Вы находитесь на странице: 1из 369

CHAPTER 1

Introduction

Concrete
Concrete is a mixture of water, cement, sand, gravel crushed rock, or other aggregates.
The aggregates (sand, gravel, crushed rock) are held together in a rocklike mass with a
paste of cement and water.

REINFORCED CONCRETE
As with most rocklike mass, concrete has very high compressive strength but have a
very low tensile strength. As a structural member, concrete can be made to carry
tensile stresses (as in beam in flexure). In this regard, it is necessary to provide steel
bars to provide the tensile strength lacking in concrete. The composite member is
called reinforced concrete.

AGGREGATES
Aggregates used in concrete may be fine aggregates (usually sand) and coarse
aggregates (usually gravel or crushed stone). Fine aggregates are those that passes
through a No. 4 sieve (about 6 mm in size). Materials retained are coarse aggregates.
The nominal maximum sizes of coarse aggregate are specified in Section 5.3.3 of
NSCP. These are follows: 1/5 the narrowest dimension between sides of forms, 1/3 the
depth of slabs, or 3/4 the minimum clear spacing between individual reinforcing bars or
wires, bundles of bars, or prestressing tendons or ducts. These limitations may not be
applied if, in the judgment the Engineer, workability and methods of consolidation are
such that concrete can be placed without honeycomb or voids.

WATER
According to Section 5.3.4, water used in mixing concrete shall be clean and free from
injurious of oils, acids, alkalis, salts organic materials or other substances that may be
deleterious to concrete or reinforcement. Mixing water for prestressed concrete or for
concrete that will contain aluminum embedment’s, including that portion of mixing water
contributed in the form of free moisture on aggregates, shall not be used in concrete
unless the following are satisfied: (a) Selection of concrete proportions shall be based
on concrete mixes using water from the same source and (b) mortar test cubes made
with non-portable mixing water shall have 7-days and 28 day strengths equal to at least
90
MODULUS OF ELASTICITY
Unlike steel and other materials, concrete has no definite modulus of
elasticity. Its value is dependent on the characteristics of cement and
aggregates used, age of concrete and strengths.

According to NSCP (Section 5.8.5), modulus of elasticity E c for concrete for


values of wc, between 1500 and 2500 kg/ m3 may be taken as.

0.043 √ f c
'
EC =W c
1.50
Eq. 1-1
(¿ MPa )

f'c Wc
Where is the day 28-day compressive strength of concrete in MPa

is the unit weight on concrete in kg / m3 . For normal weight concrete,

E c = 4700 √ f ' c . Modulus of elasticity E for nonprestressed reinforced may


s

be taken as 200,000 MPa.

DETAILS OF REINFORCEMENT

STANDARD HOOKS

Standard hooks refer to one of the following:


4 db
1. 180-degree bend plus extension but not less than 60 mm at free end
of bar.
12 d b
2. 90-degree bed plus extension at free end of bar.
3. For stirrups and tie hooks:
6 db
a) 61 mm diameter bar and smaller, 90-degree bend plus
extension at
free end bar, or
12 db
b) 20 and 25 mm diameter bar, 90-degree bend, plus extension
at free end of bar, or
6 db
c) 25mm diameter bar and smaller, 135-degree bend d plus
extension at free end of bar.

MINIMUM BEND DIAMETERS (SECTION 407.3)


Diameter of bend measured on the inside of the bar, other than for stirrups and ties in
sizes 10mm through 15 mm, shall not be less than the values in Table 1.1.

4 db
Inside diameter of bend for stirrups and ties shall not be less than 16 mm bar and
smaller. For bars larger than 16 mm, diameter of bend shall be in accordance with Table
1.1

Inside diameter of bend in welded wire fabric /9plain or deformed) for stirrups and ties
4 db 2 db
shall not be less than for deformed wire larger than D56 and for all
other
wires.
4 db
Bends with inside diameter of less than 8db shall not be less than from
nearest welded intersection.

Table 1.1- Minimum Diameters of Bend

Bar Size Minimum Diameter


6 db
10 mm to25 mm
8 db
28 mm, 32 mm, and 36 mm

PLAIN REINFORCEMENT (407.6)


Reinforcement, prestressing tendons, and ducts shall not be accurately placed and
adequately before concrete is placed, and shall be secured against displacement within
tolerance permitted.
Unless otherwise specified by the Engineer, reinforcement prestressing tendons, and
prestressing ducts shall be placed within the following tolerances:

Tolerance for depth d, and minimum concrete over a flexural members walls and
compression members shall be as follows:

Effective depth, d Tolerance on d Tolerance on minimum


concrete cover
≤200 mm ±10 mm
d -10 mm

¿ 200 mm ±12 mm
d -12 mm

Except that tolerance for the clear distance to formed soffits shall be minus 6 mm and
tolerance for cover shall not exceed minus 1/3 the minimum concrete cover required in
the design drawings or specifications.

±
Tolerance for longitudinal location of bends and ends of reinforcement shall be 50
±12
mm except at discontinuous ends of members where tolerance shall be mm.

SPACING LIMITS FOR REINFORCEMENT


According for Section 5.7.6 of NSCP, the minimum clear spacing between parallel bars
in a layer should be db but not less than 25 mm. Where parallel reinforcement is placed
in two or more layers, bars in the upper layers should be placed directly above bars in
the bottom layer with clear distance between layers not less than 25mm. In spirally
reinforced or tied reinforced compression members, clear distance between longitudinal
bars shall be not less than 1.5 db nor 40mm.

In walls and slabs other than concrete joist construction, primary flexural reinforced
shall be spaced not for farther apart than three times the wall or slab thickness, nor 450
mm.

BUNDLED BARS
Groups of parallel reinforcing bars bundled in contact to act as unit shall be limited to
four in any one bundle. Bundled bars shall be enclosed within stirrups or ties and bars
larger than 32 mm shall not be bundle in beams. The individual bars within the span of
flexural members should terminate at different points with at least 40 db stagger. Since
spacing limitations and minimum concrete cover of most members are based on a
single diameter db, bundled bars shall be treated as a single bar of a diameter derived
from the equivalent total area.

Figure 1.1 Bundled-bar arrangement

Diameter of single bar equivalent to bundled bars according to NSCP to be used for
spacing limitation and concrete cover.

3-25mm Equivalent diameter, D


π π
4 (25) 2
x3 4 D2
CONCRETE PROTECTION FOR REINFORCEMENT (SECTION 407.8.1)

Cast-in –place Concrete (nonprestressed). The following minimum concrete cover


shall be provided for reinforcement:
Minimum
cover, mm
(a) Concrete cast against permanently exposed to earth 75

(b) Concrete exposed to earth or weather: 50


20 mm through 36 mm bars 40
16 mm bar, W31 or D31 wire, and smaller
(C Concrete not exposed to weather or in contact with
) ground: 20
slabs, walls, joists:
32 mm bar and smaller 40
Beams, columns
Primary reinforcement, ties, stirrups, spirals 20
Shells, folded place members:
20 mm bar and larger 15
16 mm, Wr1 or D31 wire, and smaller

Precast concrete (Manufactured Under Plant Conditions).The Following minimum


concrete shall be provided for reinforcement

Minimum
cover, mm

(a) Concrete exposed to earth or weather:


Wall panels: 20
32 mm bar and smaller
Other members: 40
20 mm through 32 mm bars 30
16 mm bar, W31 wire, and smaller

(b) Concrete not exposed to weather or in contact with


ground: 15
slabs, walls, joists: db but not less 15, &
32 mm bar and smaller need not exceed 40
Beams, columns 10

Primary reinforcement
15
Ties, stirrups, spirals 10
Shells, folded plate members:
20 mm bar and larger
16 mm, Wr1 or D31 wire, and smaller

Prestressed Concrete
The following minimum concrete cover shall be provided for prestressed and
nonprestressed reinforcement, ducts and end fittings.

Minimum
cover, mm

(a) Concrete cast against permanently exposed to earth 75

(b) Concrete exposed to earth or weather:


Wall panels, slabs joists 25
other members 40

(C) Concrete not exposed to weather or in contact with


ground: 20
slabs, walls, joists:
Beams, columns: 40
Primary reinforcement,
25
Ties, stirrups, spirals
Shells, folded plate members: 10
16 mm, Wr1 or D31 wire, and smaller
db but not less than
Other Reinforcement 20

Bundled Bars
For bundled bars, the minimum concrete cover shall be equal to the equivalent
diameter of the bundle, but need to be greater than 50 mm, except for concrete cast
against and permanently exposed to earth, the minimum cover shall be 75 mm.

SHRINKAGE AND TEMPERATURE REINFORCEMENT (2010 NSCP)

Shrinkage and temperature reinforcement is required at right angles to the principles


reinforcement to minimize cracking and to tie the structure together to ensure its acting
as assumed in the design. The provisions of this section are intended for structural
slabs only; they are not intended for soil-supported slabs on grade.

Reinforcement for shrinkage and temperature stresses normal to flexural reinforcement


shall be provided in structural slabs where the flexural reinforcement extends in one
direction only.

Shrinkage and temperature reinforcement shall be provided in accordance with either of


the following:

a) Where shrinkage and temperature movements are significantly restrained,


the requirements of 408.3.4 and 408.3.3 shall be considered.
b) Deformed reinforcement conforming to 43.6.3 used for shrinkage and
temperature reinforcement shall be provided in accordance with the
following:
Areas of shrinkage and temperature reinforced shall be provided at least the following
rations of reinforcement area to gross concrete area, but no less than 0.014:
a) Slabs where Grade 280 or 350 deformed bars are used…….0.0020
b) Slabs where Grade 420 deformed bars or welded wire reinforcement are
used..................................................................................................0.0018
c) Slabs where reinforcement with stress exceeding 420 MPa measured at a
yield strain of 0.35 percent
is used…………………………………………………….....
f
…..0.0018x415/ y

Shrinkage and temperature reinforcement shall be spaced not farther apart than five
times the slab thickness, nor farther apart than 450 mm.

LOADS
The most important and most critical task of an engineer is the determination of the
loads that can be applied to a structure during its life, and the worst possible
combination of these loads that might occur simultaneously. Loads on structure may be
classified as dead loads or live loads.

DEAD LOAD
Dead loads are loads of constant magnitude that remain in one position. This consists
mainly of the weight of the structure and other permanent attachments to the frame .

LIVE LOAD
Live loads are loads that may change in magnitude and position. Live loads that move
under their own power called moving loads. Other Live loads are those caused by wind,
rain, earthquakes, soils, and temperature changes. Wind and earthquake loads are
called lateral loads.

ARRENGMENTS OF LIVE LOAD


Live loads may be applied only to the floor or roof under consideration, and the far ends
of columns built integrally with the structure may be considered fixed. It is permitted by
the code to assume the following arrangement of live loads:
(a) Factored dead load on all spans with full factored live load on two adjacent
spans, and
(b) Factored dead load on all spans with full factored live load on alternative
spans.
REQUIRED STRENGHT (FACTIRED LOAD), U
Required strength U to resist dead load (D) and live load (L) shall be at least equal
to: Eq. 1-2 U=1.4D + 1.7L

If resistances to structural effects of a specified wind load W are included in design, the
following combination of D, L, and W shall be investigated to determine the greatest
required strength U:

Eq. 1-3 U=0.75(1.4D + 1.7L + 1.7W)

Where load combinations shall be include both full value and zero value of L to
determine the more severe condition, and

Eq. 1-4 U=0.9D + 1.3W

But for any combination of D, L, and W, required strength U shall not be less than Eq.1-
2

If resistance to specified earthquake loads of forces E is included in design, the


following combinations of D, L and E shall be investigated to determine the greatest
required strength U:

Eq. 1-5 U=1.1D + 1.3L + 1.1E

Where load combinations shall included both full value and zero value of L to determine
the more severe condition, and

Eq. 1-6 U=0.9D + 1.1E

But for any combination of D, L, and E, required strength U shall not be less than Eq. 1-
2

If resistance to earth pressure H is included in design, required strength U shall be at


least equal to:
Eq. 1-7 U=1.4D + 1.7L + 1.7 H

Except where D or L reduces the effect of H, 0.9D shall be substituted for 1.4D and
zero value of L shall be used to determine the greatest required strength U. For any
combination of D, L and H, required strength U shall not be less than.

If resistance to loadings due to weight and pressure of fluids with well defined densities
and controllable maximum heights F is included in design, such loading shall have a
factor of 1.4 and to be added to all loading combinations that include live load.

If resistance to impact effects is taken into account in design, such effects shall be
included with live load L.

Where structural effects T of differential settlement, creep, and shrinkage expansion of


shrinkage-compensating concrete or temperature change may be significant in design,
required strength U shall be equal to

Eq. 1-8 U=1.75(1.4D +1.4T + 1.7L)

But required strength U shall not be less than

Eq. 1-9 U=1.4(D + T)

Estimations of differential settlement, creep, and shrinkage expansion of shrinkage


compensating concrete or temperature change shall be based on a realistic
assessment of such effects occurring in service.

STRENGTH REDUCTIONS FACTORS, φ (PHI)

The design strength provided by a concrete member, its connections to other members,
and its cross sections, in terms of flexure, axial load, shear, and torsion shall be taken
as the nominal strength multiplied by a strength reduction factor φ having following
values.

(a) Flexure without axial load.....................................................0.90


(b) Axial tension, and axial tension with flexure.......................0.90
(c)Axial tension and axial tension with flexure:
1. Spiral reinforcement...........................................................0.75
2. The reinforcement & other reinforced members…........0.75

(d) Shear and torsion.......................................................................0.85


(e) Bearing on concrete...................................................................0.70
(f) Post-tensioned anchorage zones…........................................0.85

ACI-318-05 (NSCP C101-10-210)

Notations
A g=¿ Ag
gross of concrete sections , mm . For a hollow section,
2
is the area
of the concrete only and does not include the area of the void(s)

A v =¿
area of shear reinforcement spacing, mm
2

bw=¿
web width, or diameter of circular section, mm

D = dead loads, or related internal moments and

forces

d = distance from extreme compression fiber to centroid of longitudinal


tension reinforcement, mm

E = load effects of earthquake, or related internal moments and forces

f yt =¿ fy
specified yield strength of transverse reinforcement, MPa

F = loads due to weight and pressures of fluids with well-defined densities and
controllable maximum heights, or related internal moments and forces.

h = overall thickness or height of member, mm


H = loads due to weight and pressure of soil water in soil, or other materials, or
related internal moments and forces.

L = live loads or related internal moments and forces.

Lr =¿
roof live loads or related internal moments and forces.

M u =¿
factored moment at section, N-mm

Nu=¿ Vu
factored axial force normal to cross section occurring simultaneously with
Tu
or ; to be taken as positive for compression and negative for tension, N

R = rain load, or related internal moments and forces.

T = cumulative effect of temperature, creep, shrinkage , differential settlement,


and shrinkage-compensating concrete.

U = required strength to resist factored loads or related internal moments and forces,

Vc
= nominal shear strength provided by concrete, N

Vn
= nominal shear strength.

Vs
= nominal shear strength provided by shear reinforcement N

Vu
= factored shear force at section, N

W = wind load, related internal moments and forces

εt
= net tensile strain in extreme layer of longitudinal tension steel at nominal
strength, excluding strains due to effective prestress, creep, shrinkage,
and temperature
φ = strength reduction factor

ρw As bw d
= ratio of to

CHAPTER 9 – STRENGTH AND SERVVICEABILITY REQUIREMENTS

9.1- GENERAL
9.1.1 Structures and structural members shall be designed to have design
strengths at all sections at least equal to the required strengths calculated for the
factored loads and forces in such combinations as are stipulated in this code.

9.1.2 Members also shall meet all other requirements of this code to ensure
adequate performance at service load levels.

9.1.3 Design of structures and structural members using the load factor
combinations and strength reduction factors of Appendix C shall be permitted.
Use of load factor combinations from this chapter in conjunction with strength
reduction factors of appendix C shall be permitted.

9.2 Required strength


9.2.1 Required strength U shall be at least to the effects of factored loads in Eq.
(9-1) through (9-7). The effect of one or more loads not acting simultaneously
shall be investigated.

U = 1.4 (D+F) (9-1)


lR
U = 1.2(D+F+T) + 1.6(L+H) + 0.5( or R) (9-2)
Lr
U = 1.2D + 1.6( or R) + (1.0L or 0.8W) (9-3)
Lr
U = 1.2D + 1.6W + 1.0L + 0.5( or R) (9-4)
U = 1.2D + 1.0E+ 1.0L (9-5)
U = 0.9D + 1.6W+ 1.6H (9-6)
U = 0.9D + 1.0E+ 1.6H (9-8)

Except as follows:

a) The load factor on the live load L in Eq. (9-3) to (9-5) shall be permitted to be
reduced to 0.5 except for garages, areas occupied as places of public assembly,
and all where L is greater than 4.8N/ m .
2

b) Where wind load W has not been reduced by a directionality factor, it shall be
permitted to use 1.3 W in Eq. (9-4) and (9-6).
c) Where E, the load effects of earthquake, is based on service-level seismic
forces, 1.4E shall be used in place of 1.0E Eq. (9-5) and (9-7).
d) The load factor on H, loads due to weight and pressure of soil, water in soil or
other materials, shall be set equal to zero in Eq. (9-6) and (9-7) if the structural
action due to H counteracts that due to W or E. Where lateral earth pressure
provides resistance to structural actions from other forces, it shall be not be
included in H but shall be included in the design resistance.

9.2.2 If resistance to impact effects is taken into account id design, such effects
shall be included with L.

9.2.3 Estimations of differential settlement, creep, shrinkage, expansion of


shrinkage-compensating concrete. or temperature change shall be based on a
realistic assessment of such effects occurring in service.

9.2.4 If structure is in a flood zone, or is subjected to forces from atmospheric ice


loads, the flood or ice loads and the appropriate load combinations of
SEI/ASCE7 shall be used.

9.2.5 For post-tensioned anchorage zone design, a load factor of 1.2 shall be
applied to the maximum prestressing steel jacking force.

9.3 Design strength


9.3.1 Design strength provided by a member, its connections to other members,
and its cross sections, in terms of flexure, axial load, shear and torsion, shall be
taken as the nominal strength calculated in accordance with requirements and
φ
assumptions of this code, multiplied by the strength reduction factors
in
9.3.2,9.3.4, and 9.3.5.

φ
9.3.2 Strength reduction factor shall be as given in 9.3.2.1 through 9.3.2.7:
9.3.2.1 Tension-controlled sections as defined in 10.3.4................0.90
(See also 9.3.2.7)

9.3.2.2 Compression-controlled sections, as defined 10.3.3:


a) Members with spiral reinforcement conforming to 10.9.3…..0.70
b) Other reinforced members…......................................................0.65

For sections in which the net tensile strain in the extreme tension steel at nominal
εt
strength is between the limits for compression-controlled and tension-
controlled
φ
sections, shall be permitted to be linearly increase from that for compression-
limit to 0.005.
( )
( )
0.9
0 Spiral
( )
0 ( )
0.
7
0.6 5O th e r fy
Alternativel ,w he n Appendix B is used, for does not exceed
y
members in which

415 MCPao,
mwitphrseymssmieoTtrnricarneisnfiotricoemnenTt,eanndsiwoitnh
φ
(d-d’)/h not less than 0.70, shall
eased linearly toc0o.9n0tarsoll e dn decreasesf 'from
φ P
c Ag
controlled
be pe 0.10 to
rmitted to be incr
F i g u r e 1 . 2 S t re n g t h r e duφ
zero. F o r o th e r r ei n for ce d m e m b er s ,
ct i o n f a c t o r
s h a ll b e p e r m itted to be increased from 0.10
(E=200, 000 MPa)
f ' c Ag φ Pb
or , whichever is smaller, to zero.

9.3.2.3 –Shear and torsion...............................................................0.75


9.3.2.4 – Bearing on concrete (except for post-tensioned and anchorage
zones and struct-and-tie models)…................................................0.65

CHAPTER 1

Analysis and Design of Beam


NOTAIONS AND SYMBOLS USED
∂ = depth of equivalent stress block, mm
As
 = area of tension reinforcement, mm2
A sk
= area of skin reinforcement per unit height in one side face, mm2/ m
b  = width of compression face of member, mm

c  = distance from extreme compression fiber to neutral axis, mm

d = distance from extreme compression fiber to centroid of


tension reinforcement, mm
d' =distance from extreme compression fiber to centroid of
compression reinforcement, mm
dc
= thickness of concrete cover measured from extreme tension fiber to center of
bar or wire, mm
Ec
=modulus of elasticity of concrete, MPa
Es
= modulus of elasticity of steel 200,000 MPa
f 'c
 =specified compressive stress of concrete, MPa
fs
=calculated stress in reinforcement at service loads, MPa
fy
=specified yield strength of steel, MPa
h =overall thickness of member, mm
 

Ig
=moment of inertia of gross concrete section about centroidal axis,
neglecting reinforcement
I se
=moment of inertia of reinforcement about centroidal axis of member
cross- section
Mn
=nominal moment, N-mm
Mu
=factored moment at section, N-mm
β1
=factor defined in Section 410.4 in Page 16
εc
=strain in concrete (maximum = 0.003)
εs f y / Es
=strain in steel below yield point =
εy
=strain in steel at yield point
ρ As
=ration of tension reinforcement
/bd ρb

=balance steel ratio


Ø =strength reduction factor

ASSUMPTION IN STRENGTH DESIGN IN FLEXURE


(CODE SECTION 5.10.2)
1. Strain in reinforcement and concrete shall be based assumed directly proportional to
the distance from the neutral axis. Expect for deep flexural members with overall depth
to clear span to ratio, h/L> 2/5 for continuous spans and h/L >4/5 for simple spans, a
nonlinear distribution of strain shall be considered (See Sec. 5.10.7).

Ec
2. Maximum usable strain at extreme concrete compression fiber, shall
be assumed equal to 0.003

fs fy fs εs εs εs εs fs
3. For below shall , be taken as x for > , =
fy
.

4. Tensile strength of concrete shall be neglected in axial and flexural calculations.


5. Relationships between compressive stress distribution and concrete strain may be
assumed rectangular, trapezoidal, parabolic, or any other from that result in prediction
of strength in substantial agreement with results of comprehensive tests.

6. For rectangular distribution of stress:

f'c
a) Concrete stress of 0.85 shall be assumed uniformly distributed over an
equivalent compression zone bounded by edges of the cross-section and a
straight line located parallel to the maximum compressive strain.
b) Distance c from fiber of maximum strain to the neutral axis hall is measured in
the direction perpendicular to N.A.
β1 f'c β1
c) Factor shall be taken as 0.85 for ≤ 30 MPa and shall be

reduced continuously at rate of 0.008 for each 1 MPa of strength in excess of 30


β1 but
MPa,
 shall not be taken less than 0.65. i.e

f'c β1
i. For ≤ 30 MPa, = 0.85

f'c
ii. For  > 30 MPa,
β1  =0.85-0.008( f ' -30) but not shall be less than 0.65
c

RECTANGULAR BEAM REINFORCED FOR TENSION ONLY


(SINGLY REINFORCED)

f'c
b 0.85 0.003

c a
c

d d-
a/2 Mn
N
A
As f y
T=
y/¿
Es f
¿

Stress
Diagram
Strain
Diagram
Figure 2.1: Stress and strain diagram for singly reinforced and rectangular beam

a=β1 c
Eq. 2-1

f ' c ≤ 30 MPa, β1=0.85


For
0.05
= −
f ' c ≤ 30 MPa, β 1 0.85 f ' c−30 but shall not be less than 0.65
For 7
( ¿

[ ∑ Fh =0 ] C=T
f ' c ab= A s f y
0.85

As f y
a=
Eq. 2-2 0.85 f ' c b

Multiplying Eq. 2-2 by d/d:


As f y
a= c
dx 0.85 f ' b
d

As f y d
a = bd 0.85 f '
c
As
The ρ
bd is called the ratio of steel reinforcement and is denoted as .
term

As
Eq. 2-3 ρ=
bd
and

p f yd
a=
Eq. 2-4 0.85 f ' c

ρf y
Let ω=
f 'c

ωd
Eq. 2-5 a=
0.85

Nominal Moment Capacity:

From the stress diagram in Figure 2.1:


M n =Cx( d−a /2)

Mn=0.85 f ' c ab (d−a/ 2)


' ωd 1 ωd
M =0.85 f b( d− )
n c
0.85 2 0.85
Eq.2-6 Mn=f c' ωbd (1−0.59
2
ω)

Ultimate Moment Capacity (Design Strength):


M u =φ M u (where φ=0.90 for flexure )

Eq.2-7 Mu=φ f 'c ωb d2 (1−0.59 ω)

Coefficient of Resistance
Eq.2-8 Rn=f 'c ω (1−0.59
ω)
Eq.2-9 Mu=φ Rn bd 2
ρf y
Solving for ω
in Eq. 2-8 and replacing it f ' c , yields the following formula
an with,
ρ
the steel ratio :

[ √ ]
0.85 f ' c
Eq.2-10 ρ= 1− 2 Rn
y
1−
f 0.85 f 'c

BALANCE DESIGN
Balance design refers to a design so proportioned that the maximum stresses in concrete
fy
(with strain of 0.003) and steel and (with strain of E ) are reached simultaneously once he
s

ultimate load is reached, causing them to fail simultaneously.

UNDERREINFORCED DESIGN
Underreinforced design is a design in which the steel reinforced is lesser than what is
required
for balance condition . If the ultimate load is approached, the steel will begin to yield although
the compression concrete is still understressed. If the load is further increased, the steel will
continue to elongate, resulting in appreciable deflections and large visible crack in the tensile
concrete. Failure under this condition is ductile and will give warning to the user of the
structure to decrease the load.

OVERREINFORCED DESIGN
Overreinforced design is a design in which the steel reinforcement is more than what is
required for balanced condition. If the beam is overreinforced, the steel will not before failure.
As the load is increased, deflections are not noticeable although the compression concrete is
highly stressed, and failure occurs suddenly without warning to the user of the structure.

Overreinforced as well as balanced design should be avoided in concrete because of its


brittle property, that is why the Code limits the tensile steel percentage (P max=0.75pb) to ensure
underreinforced beam with ductile type of failure to give occupants warning before occurs.

ρb
BALANCED STEEL RATIO :
In balanced condition, the concrete and steel yield simultaneously, In this condition, the strain
fy
in concrete reached is maximum usable value of and the strain in steel is where Es
Es
=200,000 MPa.

b 0.00
3
c
d = c
NA d
0.00
3

Strain Diagram

Figure 2.2-Balanced condition


By ratio and proportion in the triangle shown in Figure2.2:

c 0.003
d= fy
0.003+ Es =200,00
Es Note:

0.003
c= fy d
0.003+
200,00

600 d
c b=
Eq.2-11 600+ f y

β1
But a =
c

ρf y d
a 0.85 f 'c
c= c= β
β1
1

p f yd
c=
0.85 f ' cβ 1

p f yd 600
c= 0.85 f 'c β1 =
c
d
600+f y

0.85 f ' c β1 600


Eq. 2-12 ρb=
f y(600+ f y)

Note: Eq. 2-12 is for singly reinforced rectangular sections only. Eq. 2-11 is applicable to
nay shape.
MAXIMUM STEEL REINFORCEMENT
Section 410.4.3: For flexural and for subject to combined flexure and compressive
φ Pn
axial load when the design axial load strength is
0.10 f ' c A g
less than the smaller of
φ Pn ρb
or , the ratio of reinforcement that would produce balance strain condition
for the section under flexure without axial; load. For members with compression
ρb
reinforcement, the portion of equalized by compression reinforcement need not
be
reduced by the0.75 factor.
ρmax =0.75 ρb
Eq. 2-13

and

Asmax =0.75 ρb
Eq. 2-14

This limitation is to ensure that the steel reinforcement will yield first to ensure ductile
failure.

MINIMUM REINFORCEMENT OF FLEXURAL MEMBERS

410.61 At very section of flexural members where tensile reinforcement is required by


As
analysis, the area provided shall not be less than that given by:

A =
√ fc b d
Eq. 2-15 smin w
'4 f y

1.4 bw d
Eq.2-16 and not less than fy
Asmin
410.62 For statically determinate T-section with flange in tension, the area
shall be equal to or greater than the smaller value given either by:

A smi n=
√ f 'c d
Eq. 2-17 2 fy w

b
bw
or Eq. 2-15 with set equal to the width of the flange.

410.6.3 The requirements of Sections 410.6.1 and 410.6.2 need to be applied if at


every section the area of the tensile reinforcement is at least one-third greater than that
required by analysis.

410.6.4 For structural slabs and footings of uniform thickness, the minimum area of
tensile reinforcement in the direction of span shall be the same as that required by
Section 407.13 (Shrinking and Temperature Reinforcement). Maximum spacing of
this reinforcement shall not exceed three times the thickness and 450 mm.

The provision for minimum amount of reinforcement applies to beams, which for
architectural and other reasons are much larger in cross-section than required by
strength consideration. With a very small amount of tensile reinforcement, the
computed moment strength as a reinforced concrete section computed from its
modulus of rapture. Failure in such a case can be quite sudden.

STEPS IN DESIGNING A SINGLY REINFORCED


RECTANGULAR BEAM FOR FLEXURE:

Note: The assumptions made in steps II, V,and VIII are the author’s recommendation
based on his experience.

I. Identify the values of the dead load and live load to be carried by the
beam. (DL & LL)
II. Approximate the weight of beam (DL) between 20% to 25% of
(DL+LL).This weight is added to the de load.
III. Compute the factored load and factored
moment: Ex: factored Load =1.4 DL+1.7L
Mn
IV. Compute the factored moment to be resisted by the beam,
ρ ρb
V. Try a value of steel ratio from 0.5 but must not be less
ρmin ρ
than . This value will provided enough alloance in the actual
ρ
value of due to rounding-off of the number of bars to be
used, for it not to
ρ
exceed the maximum of 0.05ρb.
0.85 f ' c β1 600
ρb=
f y(600+ f )

β1=0.85 for f 'c ≤30 MPa

β 1=0.85−00−.008 ( f '' c −30 ) for f c >30 MPa


ρmin
=
0.4
f
y

ρf y
VI. Compute the value of ω, ω=
f'c
Solve for bd :
2
VII.
M u=φ f c bd ( 1−0.59
' 2

ω)
¿
¿
¿
¿
¿
¿
¿
2
bd =¿¿

VIII. Try ratio b/ d❑ ( from d=15b to d=2b), and solve for d, (round-off this
value to reasonable dimension). Check also the minimum thickness of
beam required by the Code a given in Table 2.1 in page 36.
After solving for d, substitute its value to Step VII, and solve for b.
Compute the weight of the beam and compare it to the assumption made
in Step II.

IX. Solve for the required steel area and number of bars.
As =pbd

Number of bars(diameter = D)
π 2 A
4D x number of bars = s

AS
STEPS IN COMPUTING THE REQUIRED TENSION STEEL AREA OF A
MU
BEM WITH KNOWN MOMENT NT AND
OTHER PROPERTIES:

Pmax ∧ Mumax
I. Solve for
Pmax =0.75 pb

P =0.75 '
0.85 f c β1 (600)
f y(600+ f y)
max

¿
¿
¿
¿
ρf
ω= y = ¿
¿
fc
M u max=φ f c ωbd (1-0.59 ω ¿
2

M u = M umax
if design as singly reinforced (Step II)
M u = M umax
if design as doubly reinforced (Step III)

ρ
II. Solve for :
2
Mu=φ Ru bd
¿
¿
¿
¿
Ru =¿ ¿
2 Ru
1−
0.85 f ' c
¿
¿
¿
1−√¿=¿ ¿
0.85 f ' c
ρ= fy ¿
¿
¿
¿
¿
As =ρbd =¿ ¿

III. Compression reinforcement is necessary. (See Chapter 3)

MU
STEPS IN COPUTING OF A BEAM WITH KNOWN TENSION STEEL AREA
AS
AND OTHER BEAM PROPERTIES:
As
I. Solve ρ ρ=
for : bd

ρb
II. Check if steel yields by computing
ρb= '
0.85 f c β1 (600)
f y (600+ f y )

ρ ≤ρb
III. `
ω=ρ f y / f c

φMu=φ f 'c ωb d2(1−0.59 ω)

ρ ≤ρb
if ,steel yields, proceed to III
ρ ≤ ρb
if ,steel does not yield, proceed to step IV.
ρ ≤ρ min As
Note: if ,the given is not adequate for the beam dimension.

IV. ρ > ρb
b 0.85 0.00
ab
c=0.8 3
5
a c
d

d-a/2

d-c

=200,0
00
T=

fs Es
Solve for from the strain diagram: [Note: =200,000MPa]
f s
Es
0.003
d−c = c
d−c
f s=600
Eq. 2-18 c

Σ F H =0 ¿
[ T=C
A s f s=0.85 f c ab β1 c
but a=
d−c '
A 600 =0.85 f (β c)b
s c 1
c
600 A s (d−c)=0.85 β1 f c b c 2

fs
Solve c by quadratic formula and solve for and a:
d−c
f s=600 ; a=β c1
c

a
2
s s
(d− 2 )
or
a
2 c
(d −2 )

MINIMUM THICKNESS OF FLEXURAL MEMBERS


According to Section 5.9.5 of NACP, minimum thickness stipulated in Table 2.1 shall
apply for one-way construction not supporting are attached to portions or other
construction likely to be damaged by large deflections, unless computation of deflection
indicates a lesser thickness can be used without adverse effects.
Table 2.1 MINIMUM THICKNESS OF NON-PRESTRESSED BEAMS OR ONE-WAY
SLABS UNLESS DEFLECTIONS ARE COMPUTED *

Minimum thickness, h
Simply One end Both ends Cantilever
supported continuous continuous
Members not supporting or attached to partitions or other construction
Member likely to be damaged by large deflections
Solid one- L/20 L/24 L/28 L/10
way slabs
Beams L/16 L/18.5 L/21 L/8

or ribbed one-

Span length L is in millimeters


Values given shall be used directly for members with normal density concrete (
ω=2300 kg /m3 ) and grade 415 reinforcement. For other conditions, the values shall
be modified as follows:

(a) For structural lightweight concrete having weights in the range 1500-2000
3
kg / m
ωc
values shall be multiplied by (1.65-0.005 ) but not less than 1.09, where
ωc
is the unit mass in kg / m3 .

fy f y /700 ¿ .
(b) For other than 415 MPa, the values shall be multiplied by (0.4 +

BEAM DEFLECTION (SECTION 5.9.5

Sect. 5.9.5.2.2 Where deflections are to be computed, deflections that occur


immediately on application of load shall be computed by usual methods or formulas for
elastic deflections, considering effects of cracking and reinforcement on member
stiffness.
Sect. 5.9.5.2.3 Unless stiffness values are obtained by a more comprehensive analysis,
Ec
immediate deflection shall be computed with the modulus of elasticity for
concrete
Ig
and with the effective moment of inertia as follows, but not greater than .

1−
[ ]M cr
Ma

[ ]
] I cr
Eq.2-19 M cr
3

I c= I +¿
Ma g

Wher
e
M cr f rI g
Y
= t

Fr
= modulus of rapture of concrete, MPa, for normal weight
f r=0.7 √f ' c
Concrete
Ma
= maximum moment in member at stage deflections is computed.
Ig
= moment of inertia of gross concrete section about centroidal axis, neglecting
reinforcement.
I cr
= moment of inertia of cracked section transformed to concrete
Y1
= distance from centroidal axis of gross section, neglecting reinforcement, to
extreme fiber in tension.

When Lightweight aggregate is used, one of the following modifications shall apply:
f
(a) When ct
is specified and concrete is proportioned in accordance with Sec. 5.5.2,
fr
shall be modified by substituting 1.8
f ct
for √ f 'c but the value of 1.8
f ct

shall not exceed √ f 'c .


f ct fr
(b) When is not specified, shall not be multiplied by 0.75 for “all
lightweight” concrete, and 0.85 for “ sand-lightweight” concrete. Linear interpolation is
permitted if partial sand replacement is used.

Sect. 5.9.5.2.4: For continuous members, effective moment of inertia may be taken as
the average of values obtained from Eq. 2-19 for the critical positive and negative
moment sections. For prismatic members, effective moment of inertia may be taken as
the value obtained from Eq. 2-19 at midspan for simple and continuous spans, and at
the support cantilevers.

Sect.5.9.5.2.5: Unless values are obtained by a more comprehensive analysis,


additional long-term deflection resulting from creep and shrinkage of flexural members
(normal weight or lightweight concrete) shall be determined by multiplying the
immediate caused by the sustained load considered, by the factor.

ε
Eq. 2-10 λ=
1+50 ρ
'

ρ '
Where shall be taken the value of reinforcement ratio for non-prestressed
compression reinforcement at midspan for simple and continuous spans,a nd at support
ε
for cantilevers. It is permitted to assume the time-dependent factor for sustained
loads to be equal to:
5 years or more...............................2.0
12 months…......................................1.4
6 months….........................................1.2
3 months…...........................................1.0
Deflection computed in accordance with Sec. 5.9.5.2.2 through Sec.5.9.5.2.5 shall not
exceed limits stipulated in Table 2.2.

Table 2.2: Maximum Permissible Computed Deflections

Type of member Deflection to be considered Deflection limitation

Flat roofs not supporting or Immediate deflection due to L/180*


attached to nonstructural live load LL
elements likely to be damage by
large deflections
Floors not supporting or Immediate deflection due to L/360*
attached to nonstructural live load LL
elements likely to be damaged
by large deflections
Roof or floor construction That part of the total L/480**
supporting, or attached to deflection occurring after
nonstructural elements not likely attachment of non structural
to be damaged by large elements (sum of the long-
deflections time deflection due to all
Roof or floor construction sustained loads and the L/20****
supporting, or attached to immediate deflection due to
nonstructural elements not likely any additional live load)****
to be damaged by large
deflections
 Limit not intended to safeguard against ponding. Ponding should be cheated by
suitable calculations of deflections, including added deflections due to ponded
water and considering long-term effects of all sustained loads, camber,
construction tolerances, and reliability of provisions for damage.
 Limit may be exceeded if adequate measures are taken to prevent damage to
supported or attached elements.
 Long=time deflections shall be determined in accordance with Sec.5.9.5.2.5 or
Sec. Attachment of nonstructural elements. This amount shall be determined on
basis of accepted engineering, data relating to time-deflection characteristics of
members similar to those being considered.
 But not greater than tolerance provided for nonstructural elements. Limit may be
exceeded if camber is provided so that deflection minus camber does not
exceeded limit.

NSCP COEFFICICIENTS FOR CONTINUOUS BEAMS AND SLASBS

Section 5.8.3.3 of NSCP states that in lieu of frame analysis, the following approximate
moment and shears are permitted for design of continuous beams and one-way slabs
(slabs reinforced to resist flexural stresses in only one direction), provided:

a) There are two or more spans,


b) Spans are approximately equal, with the larger of two adjacent spans not greater
than uniformly than the shorter by more than 20 percent,
c) Loads are uniformly distributed,
d) Unit live does not exceeded three times unit dead load, and
e) Members are prismatic.

Positive moment
End spans
ωu Ln / 11
Discontinuous end unrestrained…………………… 2

ωu Ln / 14
Discontinuous end integral with support………….. 2

ωu Ln / 16
Interior spans………………………………… 2

Negative moment at exterior face of first interior support


ωu Ln / 9
Two spans …………………………………………………………..... 2

ωu Ln / 10
More than two spans………………………………………………..... 2

ωu Ln / 11
Negative moment at other faces of interior supports………….…… 2

Negative moment at face of all supports for:


Slabs with spans not exceeding 3 m; and beams
Where ratio of sum of column stiffness to beams
ωu Ln / 12
Stiffness exceeds eight at each end of the span…………………… 2

Negative moment at interior face of exterior


Support members built integrally with
ωu Ln / 24
Where support is a spandrel beam…………………………… 2

ωu Ln / 16
When support is a column…………………………….……..…. 2

Shear in end members at face of


1.5 ωu Ln /2
First interior support……………………………..................................... 2

ωu Ln / 2
Shear at face of all other supports……………………………………….. 2

Ln
When =clear span positive moment or shear and average of adjacent clear
spans for negative moment.

Column Column Column


1
.
1
w 1.15w 5
w
Shear
w w

Moment

-
-w w
-w
Figure 2.3: Shear and moment for continuous beam or slab with spans and
discontinuous end integral with support

Column Column Column


w
1.15 Spandrel
w
wBeam
w w 1
.
1
5 w Shear
-
w
- - Moment
-
w - - w
w w
w w

Column Column Column

1
w . w
1.15 1
w w
5 w Shear
w
w

Mome
nt

-
w - - -
w w w
Figure 2.5 Shear and moment for continuous beam or slab with more than two spans
and discontinuous end unrestrained

ACI-318-05 (NSCP C101-10-2010)

10.2 Design assumptions (410.3)


10.2.1 Strength design of members for flexure and axial loads shall be based on
assumptions given in 10.2.2 through 10.2.7, and on satisfaction of applicable conditions
of equilibrium and compatibility of strains.

10.2.2 Strain in reinforcement and concrete shall be assumed directly


proportional to the distance from the neutral axis, except that, for deep beams as
defined in 10.7.1, an analysis that considers a nonlinear distribution of strain shall be
used alternatively, it shall be permitted to use a struct-and tie model. See 10.7,118, and
Appendix A.

10.2.3 Maximum usable strain at extreme concrete compression fiber shall be


assumed equal to 0.003.

f Es
10.2.4 Stress in reinforcement below y
shall be taken as times steel
f
strain. For strains greater than that corresponding to y
, stress in reinforcement shall
fy
be considered independent of strain and equal to .

10.2.5 Tensile strength of concrete shall be neglected in axial and flexural


calculations of reinforced concrete, except when meeting requirements of 18.4.

10.2.6 The relationship between concrete compressive stress distribution and


concrete strain shall be assumed to be rectangular, trapezoidal, parabolic, or any other
shape that results in prediction of strength in substantial agreement with results of
comprehensive tests.

10.2.7 Requirements of 10.2.6 are satisfied by an equivalent rectangular


concrete stress distribution defined by the following:
f'c
10.2.7.1 Concrete stress of 0.85 shall be assumed uniformly
distributed over an equivalent compression zone bounded by edges of the cross
β
section and a straight line located parallel to the neutral axis at distance a= 1 c form
the fiber
of maximum compressive strain.

10.2.7.2 Distance from the fiber of maximum strain to the neutral axis, c ,
shall be measured in direction perpendicular to the neutral axis.

f'c β1
10.2.7.3 For between 17 and 18 MPa, shall be taken as
f'c β1
0.85. For above 28 MPa, shall not be taken less
than 0.65

10.3 General principles and requirements (410.4)

10.3.1 Design of cross sections subject to flexure or axial loads, or to combined


flexure and axial loads, shall be based on stress and strain compatibility using
assumptions in10.2.

10.3.2 Balanced strain conditions exist at a cross section when tension


fy
reinforcement reaches the strain corresponding to just as concrete
in
compression reaches its assumed ultimate strain of 0.003.

10.3.3 Sections are compression-controlled if the next tensile strain in the


εt
extreme tension steel, , is equal to or less than the compression-controlled
strain
limit when the concrete in reaches its assumed strain limit of 0.003. The compression-
controlled strain limit is the net tensile strain in the reinforcement at balanced strain
conditions. For Grade 415 reinforcement, and for all prestressed reinforcement, it shall
be permitted to set the compression-controlled strain limit equal to 0.002.

10.3.4 Sections are tension-controlled if the net tensile strain in the extreme
εt
tension steel is equal to greater than 0.005 when the concrete in compression
εt
reaches its assumed strain limit of 0.003. Sections with between the compression-
controlled strain limit and 0.005 constitute a transition region between compression-
controlled and tension-controlled sections.
Derivation: for E = 200 GPa
The beam is tension-controlled
ε fs 0.00 0.008
When = 0.005 (or =1000MPa) 3
c

d
=

0.005
c d
=
0.003 0.008
3
c=
Eq. 2-21 8
d

a=β1 c β 3 d
=
1
8

For rectangular beam:


a
( )
φ M tn =φCc d− where φ=0.90
2

φM
tn =0.90 x 0.85 f ab( d−
a
'
)
2 3
β d
φM 1
8
=0.90 x 0.85 f c x β1
tn
' 3 / 8 dxb(d − )
2

c
459 3
φ Mtn = β1 f c' bd 2(1 − β1)
Eq. 2-22 1600 16

10.3.5 For nonprestressed flexural members and nonprestressed members with


f c Ag εt
factored axial compressive load less than 0.10 steel strain at
nominal
strength shall not be less than 0.004.

10.3.5.1 Use of compression reinforcement shall be permitted in conjunction with


additional tension reinforcement to increase the strength of flexural members.

Derivation: for E =200 GPa


M nmax
Maximum steel area and when beam is singly reinforced:

εt=0.004∨f s=εtx E=800 MPa

0.003 0.007

= d

0.004

Strain diagram for minimum steel strain


c d 3
=
0.003 0.007 ; c = 7d

3
Cmax =
Eq. 2-23 7
d

For rectangular section:

Cc A s max f y=0.85 f 'c ab where a=β 1 c


T=

3
β1 x d
7
)b
ρmax bd f y=0.85 f c ¿

3 0.85 f ' c β1
Eq. 2-14 ρmax
7 fy

( )
−a
M =c c
n max 2
=0.85 f ab (−a
'
M
n c
max
)
2 β 3d

Mn max 3 17
=0.85 f x β1 dxb (d− )
c
7 2
3
1− β
14 1
Mn =¿
Eq. 2-25 51 )
max
β f ' bd 2¿
140 c c

800−f y
Eq. 2-26 φ=0.65+0.25
1000−f y

φ Pn
10.3.6 Design axial strength of compression members shall not be taken
φ Pn max
greater than computed by Eq. (10-1) or (10-2).

10.3.6.1 For nonprestressed members with spiral reinforcement conforming to


7.10.4 or composite members conforming to 10.16:

φ Pn max =0.85 φ[ 0.85 f' c ( A g − A st )+ f y Ast ] (10-1)


10.3.6.2 For non nonprestressed members with spiral reinforcement conforming
to 7.10.5:
'
φ Pn max =0.85 φ[ 0.85 f c ( A g − Ast )+ f y Ast ] (10-2)

φ Pn
10.3.6.3 For prestressed members, design axial shall not be taken
greater than 0.85 (for members with spiral reinforcement) or 0.80 (for members
with tie
φ P0
reinforcement) of the design axial strength at zero capacity .

10.3.7 Members subject to compressive axial load shall be designed for the maximum
Pu
moment that can accompany the axial load. The factored axial force at
given
Mu
eccentricity shall not exceed that given in 10.3.6. The maximum factored moment
shall be magnified for slenderness effects in accordance with 10.10
10.4 Distance between lateral supports of flexural members

10.4.1 Spacing of lateral supports for a beam shall not exceed 50 times b, the
least width of compression flange or face.

10.4.2 Effects of lateral eccentricity of load shall be taken into account in


determining spacing of lateral supports.

10.5.1 Minimum reinforcement of flexural members


10.5.1 At every section of flexural members where tensile reinforcement is
required by analysis, except as provided in 10.5.2, 10.5.3, and 10.5.4, as provided shall
not be less than that given by
√f'c √f'c
b d∨ ρ = b d (10-3)
w min
4fy w 4fy

and not less than

Asmin 1.4 b d ρ 1.4


= w
∨ min (10-3)
fy fy

Asmin
10.5.2 –For statically determinate members with a flange in tension, shall
bw
not be less than the value given by eq. (10-3), except that is replaced by
2 bw
either or the width of the flange, whichever is smaller.

STEPS IN THE DESIGN OF SINGLY REINFORCED


RECTANGULAR BEAM FOR FLEXURE
Note: The assumption made in steps II, V, and VIII are the authors recommendation
based on his experience.
I. Determine the values of loads, Dl, LL and other loads
II. Approximate the weight of beam (DL) as follows:
Small beams: 2kN/m
Medium-sized beams:
3.5kN/m Large-sixed beams:
7kN/m
or Weight of beam in kN/m=24kN/ m x beam area in m
2 2

III. Compute the factored load on different load


combinations Example: Factored Load =1.2 DL + 1.6 LL
Mu
IV. Compute the factored moment to be resisted by the beam,
ρ ρmax ρmax
V. Try a value of steel ratio from 0.7 to 0.8 but must not be
ρmin ρ
less than . This value of will provided enough allowance in the
ρ
actual value of due to rounding-off the numbers bars to be used so
ρ
that it will not exceed the maximum .

'
0.85 f β 600
c 1
ρb =
f y (600+ fy)

β1=0.85 for f 'c ≤ 28 MPa


0.05 '
( f −28 ) for f > 28 MPa
'
β =0.85−
1 c c
7
3 0.85 f 'c
ρmax =
7 fy

f
√f ' c =
1.4
ρ = if f ' > 31.36 MPa
,othewise ρ
max c min
4fy y
ρfy
VI. Compute the value ω, ω= c
of f'
φ:
VII. Solve for the reduction factor
Solve for c:
ρ
Note: For singly reinforced rectangular beam, is
directly proportional to c:

3
Cmax where Cmax = d
c=(assumed factor) x 7

The assumed factor may range from 0.7 to 0.8 as suggested in


step V.
d−c
f =600
s
c
f s ≥ 1000 MPa,tension−controlled , φ=0.90
if

f
if
s <1000 MPa ,transition , φ=0.65+0.25 f s−f y
y

Solve for bd :
2
VIII.
M u=φ f c ω b d ( 1−0.59 ω )
' 2

¿
¿
¿
¿
2
bd =¿ ¿

IX. Try a ratio d/b (from d= 1.5b to d=2b), and solve for d. (round-off this value
to reasonable dimension)

Check also the minimum thickness of beam required by the code as given
in Table 2.1 in Page 26.

After solving for d, substitute its value to Step VII, and solve for b.
Compute the weight of the beam and it to the assumption made in Step II.

X. Solve for the required steel area and number of bars.


As =pbd

Number of bars (diameter=D)


π 2 As
4 D x number of bars =

AS
STEPS IN FINDING THE REQUIRED TENSION STEEL AREA OF A
MU
BEAM WITH KNOW REQUIRE MOMENT AND
OTHER BEAM PROPERTIES

f ' cf y Mu
Given b, d, and :
ρmax φ M n max
I. Solve for and .
3 0.85 f 'c β1
ρmax =
7 fy

M n max= 140 β 1 f c bd 1− 14 β1
)
800−f y
φ=φ=0.65+0.25 y
1000−f
¿
¿
¿
¿
¿
φ Mn max =¿ ¿
M u ≤φ M n max
if design as Singly Reinforced (Step II)
M u > φ M n max
if design as Doubly Reinforced (Step V)

II. Determine if the section in tension-controlled or transition


459 ' 2 3
φ M tn= ( − )
From Eq. 2-11: 1600 β1 f c bd 1 β1
16
f s <φ M n ,tension−controlled , φ=0.90
if ,proceed to step
f s >φ M n ,transition
if region, proceed to step IV
III

III. M u=φ Rn bd 2
¿
¿
Rn =¿ ¿
2 Rn
1− '
0.85 f c
¿
¿
1−√ ¿=¿' ¿
0.85 f c
ρ= ¿
fy
√ f 'c 1.4
ρ = if φ f ' >31.36 MPa, = f
otherwise ρ
min c min
4fy y

¿
¿
¿
As =pbd =¿ ¿
IV.
√ f 'c
ρ = if φ f ' >31.36 MPa, 1.4
c = f
otherwise ρ
min min
4fy y

¿
¿
¿
As =pbd =¿ ¿

As :
Solve for c and
Mu=φCc (d−a / 2)

f s−f y
φ=0.65+0.25 =600 d−c
1000−f where s
y
c
f

a=β1 c

Cc=0.85 f ' c ab=0.85 β1 cb

[ ]
d−c
600 −f
y
c c
0.65+0.25(
)(0.85 f c β1 cb)(d −β1 )
'

1000−f 2
y

Mu =¿
¿
¿
¿
a
¿
¿
¿
c¿ β1 c =¿ ¿
As f y=0.85 f ' c ab
¿
¿
¿
¿
¿
¿
¿
¿
¿
As =¿ ¿
ρ =
√f f ' >31.36 MPa, otherwise 1.4
'c ρ =
min c min
4fy if fy

V. Compression reinforcement is necessary.(See chapter 2)

φ Mn
STEPS IN FINDING OF A BEAM WITH KNOWN TENSION STEEL AREA
AS
AND OTHER BEAM PROPERTIES:

As f'c fy
Given: b, d, , , :
As
ρ=
I. Solve for bd
ρb
II. Check if steel yields by computing
'
0.85 f c
ρ= β1(600)
b
f y (600+ f y)

ρ ≤ρ b ,
if steel yields, proceed to step III
ρ > ρb ,
if steel dos not yield, proceed to step IV.
ρ ≤ρ min , As
Note: if the given is not adequate for the beam dimension.
√ f 'c
ρmin = if f >31.36 MPa , otherwise 1.4
mi = f
' ρ n y
c
4fy
ρ ≤ρ b
III.
φ
Solve for :
As f y=0.85 f ' c ab
¿
¿ a/ β1
a=¿ ¿ ;c= =
d−c
f 600
s=
c
f s ≥ 1000 MPa,tension−controlled , φ=0.90
if

f
if
s <1000 MPa ,transition , φ=0.65+0.25 f s−f y

φ Mn a
=φ 0.85 f ' ab(d− )
c
2

IV. ρ > ρb
Compression-controlled
φ=0.65 f'c
b 0.85
f s=600 d−c
c a c=
0.85 f ' c ab

d d-a/2
T=C
As f s=0.85 f ' c ab
β1 c As f s
but a= T=
d−c
A 600 0.85 f
= c (β1 c)b
'
s
c

f s=600
c= ; d−
cc =
¿
¿
a= ¿
β1 c =¿ ¿
φ=0.65

( )
φ M n=φT d −a =φ A s f s (d− )
2 2

or
a a
( )
φ M n=φC d −2 =φ f c' ab(d −2

)
ILLUSTRATIVE PROBLEMS
DESIGN PROBLEMS

PROBLEM 2.1
A reinforced concrete rectangular beam 300 mm wide has an effective depth of 460
f ' c=21 MPa f y=345 MPa
mm and is reinforced for tension only. Assuming and ,
determine the balance steel area in sq.mm.
SOLUTION

0.85 f ' c β1600


ρ= β =0.85 since f <30 MPa
'
b 1 c
f y ( 600+f y )

0.85 ( 21) ( 0.85) (600)


ρb=
345(600+345)
ρb=0.02792

Asb= ρb bd

PROBLEM 2.2
A rectangular beam has b = 300 mm and d =490 mm. Concrete compressive strength
f ' c=27.6 MPa f y=276 MPa
and steel yield strength . Calculate the required tension
Mu
steel area if the factored moment is (a) 20 kN-m,(b)140 kN-m,(c) 485 kN-m, and
(d)620 kN-m.

SOLUTION
ρmax ∧ M u max
Solve for
:
0.85 ( 27.6) 0.85(600)
0.85 f ' c β1600 ρ=
ρ=
f y ( 600+f y )
b b
276 (600+276)

ρb=0.0495
ρmax =0.75 ρb ρmax =0.75(0.0495)

ρmax =0.0371
ρmax f y
ωmax = ωmax = 0.03711 ( 276)
f 'c 27.6

ωmax=0.371

Rn max =f ' c ω(1−0.59 ω) Rn max =27.6 (0.371)[1−0.59 (0.37)]

Rn max =8.001 MPa

490
¿
Mn bd
2
¿
n

max =R max
M n max=8.001 ( 300 ) ¿

Mn max=576.279 x 10 N −mm
6

Mn max=576.279 kN −mm

M u max=φ M n max M u max=0.90 x 576.279

Mu max=518.65 kN −m

Mu=20 kN −m < M u max=(singly reinforced )


a)
0
Mu=φ Rn bd 2 49 ¿
20 ¿
6
x 10 =0.90 R n (300)¿

Rn=0.309 MPa
√ ]
0.85 f ' 2 Rn
1−
ρ= y
c 1− 0.085 f 'c
f
0.309

[
¿
2¿
1−¿
1−√ ¿
0.85(27.6)
ρ= 276 ¿

ρ=0.00113< ρmin

√ f 'c
ρmin = if f 1.4
= f
>31.36 MPa , otherwise mi
' n y

4fy
1.4
= =0.005072
ρmi f
n
y

A s =ρbd A s =0.00572(300 )(490)

As =746 mm
2

Mu=140 kN −m< Mu max


(singly reinforced)
b)
0
M u=φ Rn bd 2 49 ¿
140 x ¿
6
1 0 =0.90 R n (300 )¿

Rn=2.16 MPa

√ ]
0.85 f ' 2 Rn
1−
ρ= y
c 1− 0.85 f 'c
f

[
[ √ ]
0.85(27.6)
ρ= 276 1− 2 ( 2.16)
1− 0.85 (27.6)
A s =ρbd ρ=0.00822>

ρmin A s =0.00822(300 )(490)

As =1,209 mm
2

Mu=485 kN−m < M umax


(singly reinforced)
c)
0
M u =φ Rn bd 2 49 ¿
485 ¿
10 =0.90 R n (300)¿
2
x

Rn=7.48 MPa

2 Rn
1−
0.85 f 'c
1−√¿
0.85 f ' c
ρ= ¿
fy

0.85(27.6)


2 (7.48) ]
ρ= [ 1 − 1−
0.85 (27.6)
276
ρ=0.03384> ρmin

A s =ρ b d A s =0.03384 (300)( 490)

As =4,975 m m
2

Mu=600 kN −m > M umax


d)

The beam will be doubly reinforced. See Chapter 3.

PROBLEM 2.3 (CE MAY 2012)


A reinforced concrete beam has a width of 300 mm and an overall depth of 480 mm.
f y=415
The beam is simply supported over span of 5 m. Steel strength MPa and
f ' c=28 MPa
concrete . Concrete cover is 70 mm from the centroid of the steel area.
3
Unit weight concrete is 23.5kN/ m .Other than the weight of the beam , the beam
carries a superimposed dead of 18 kN/m and a live load of 14 kN/m. Use the strength
design method.
Determine the maximum factored moment on the beam.
If the design ultimate moment capacity of the beam is 280 kN-m, determine the
required number of 20 mm tension bars.
If the beam will carry a factored load of 240 kN at midsoan, determine the required
number of 20 mm tension bars.

SOLUTION
f ' c=300 MPa
Given: b=300m
β1=0.85
d=480-70=410 mm
f y=415 MPa 1.4
= =0.00337
mi f
n
ρ
y
db=20 m
Bar diameter ,
k
w b=γ c A b=23.5 (0.3 x 0.48 )=3.384
Weight of beam, m

a) Maximum factored moment on the beam.


W u=1.4 (3.384+18)+.7(14)
Factored load,
Wu=53.738 kN / m
Factored load,

Maximum factored moment:

5
2 ¿¿
W L
= u
¿2
Mu 8
53.738 ¿
Mu =¿

Mu=167.93 kN −m

b) Mu=280 kN −m
M umax
Solve for to determine whether compression steel is needed

0.85 f ' c β1600


ρ= 0.85 ( 28) ( 0.85) (600)
b
f y ( 600 +fy )
ρb=
415 (600+415)

ρb=0.02881

ρmax =0.75 ρb ρmax =0.021261


ωmax ρmax f y
= ωmax =0.03203
f 'c
Rn
=f ωma (1−0.59 )=7.274
'
max
c
x
ωmax

Mu max=φ Rnmax bd =330.14 kN −m


2

Mu=280 kN −m M u max
Required < (singly reinforced)
0
41 ¿
2
¿ 6
Mu=φ Rn bd 280 x 10 =.90 R (300)¿
n

Rn=6.169 MPa

0.85 f '
R ρ=
fy c
1−
2

[
0.85 f ' c ]
ρ=
0.85(28)
415 1−
[ √ ]
2 ( 6.619 )
1− min0.85(28)=0.01755> ρ

A s =ρ b d A s =0.01755 (300)(410)

As =2159 mm
2

0
2¿
π ¿ π
A = d 2159= ¿
s
4 b 4
2

N=6.9 say 7 bars


Pu=240 kN at midspan
3.
W d =3.384 kN / m
(weight of beam)
2
P L (1.4 W ) L
u d
M = + =314.805 kN −m < (singly )
M
u u max
4 8

Mu
R=
0
41 ¿
¿
n
φb 0.90 (300)¿ 6
2 314.805 x 1 0
d
Rn =
¿
Rn=6.936 MPa

√ ]
0.85 f ' 2 Rn
1−
ρ= c 1− 0.85 f 'c
fy

0.85(28)
[
[ √ ]
ρ= 415 1− 2 Rn
1−
0.85 f 'c =002031> ρmin

A s =ρ b d A s =0.02031(300 )(410)

As =2498 mm
2

π 20¿
A = d N
¿
s
4 b2 2498 = N
π
4¿
N =7.95 say 8 bars
PROBLEM 2.4 (CE MAY 1993)
A reinforced concrete beam has a width of 300 mm and an effective depth to tension
bars of 600 mm. compression reinforcement if needed will be placed at a depth of 60
f ' c=30 MPa f y=414 MPa
mm below the top. If and , determine the tension steel
area if the beam is to resist an ultimate moment of 650 kN-m.

SOLUTION
ρmax M umax
Solve for and :
0.85 f ' c β1600
' =
ρ β =0.85 since f <10 MPa
f y ( 600+f y )
b 1 c

0.85 ( 30) ( 0.85) (600)


ρb=
414 (600+414 )
ρb=0.031

ρmax =0.15 ρb ρmax =0.75 (0.031)

ρmax =0.02323

ρf y ( 414 )
ω= ω= 0.02323
'
f c
30
ω=0.3209

Mu max=φ f 'c ωbd 2(1−0.59 ω)

0
60 ¿
¿ [1-0.59(0.309)
M u max=0.90 ( 30 ) ( 0.3209 ) ( 300 )
¿
Mu max=758.1 kN −m > Mu

Mu< Mu max
Since , the beam may be designed as singly reinforced.
0
60 ¿
Rn=6.687 MPa ¿
650 x
0 =0.90 R n (300 )¿
6

Rn=6.687 MPa

ρ
Solve for :

( √
0.85 f ' c
ρ= 1−

)
fy 2 Rn
1−
0.85 f ' c

[ √
0.85(30)
2 ( 6.687 )
]
ρ 414 1
= − 1− min0.85(30)=0.0191> ρ

1.4
ρmi = f =0.00338
n
y

A s =ρbd A s =0.0191(300 )(600)

As =3442m m
2

PROBLEM 2.5 (CE November 2000)

A rectangular concrete beam has a width of 300 mm and an effective depth of 550 mm.
The beam is simply supported over a span 6 m and is used to carry a uniform dead
f ' c=21 MPa
load of 25 kN/m and a uniform live load of 40 kN/m. Assume and
f y=312 MPa
. Compression reinforcement if necessary shall be placed at a depth 80
mm from the outermost compression concrete.
Determine 80 mm from the outermost compression concrete.
Determine the required tension steel area.
Determine the required number of 25-mm tension bars.
SOLUTION
a) Maximum steel area:

0.85 f ' c β1600


f =
ρ β =0.85 since is less than 30 MPa
b
f y ( 600+f y ) 1 c

0.85 ( 21) ( 0.85) (600)


ρb=
312(312+ 600)
ρb=0.0319
9

ρmax =0.75 ρb ρmax =0.75(0.03199)

ρmax =0.02399

A s max =ρmax bd A s max =0.02399 (300)(550)

Asmax =3,959 mm 2

b) Required tension steel

area: Factored load:


Wu=1.4 D+ 1.7 L Wu=1.4 (25)+1.7 (40)
Wu=103 kN / m

Required strength:
6
¿
¿
¿2
103 2¿
W L
M u= =¿
u
8M u

Mu
=463.5kN-m

M u max
Solve for

ρmax f y 0.0299(312)
ω= ω= 21
f 'c
ω=0.356

Mu max=φ f' c ωbd 2(1−0.59 ω)

0
55 ¿
¿
Mu max=0.90(30)(0.356)(300)¿
Mu max=536.5 kN −m > Mu singly reinforced

0
55 ¿
2
¿
Mu=φ Rn bd Mu =0.39 Rn (300 )¿
550
¿
¿
6
463.5 x 10 =0.9 Rn(300) ¿

Rn=5.67 MPa
( √ )
0.85 f ' c 2 Rn
ρ= 1− 1−
fy 0.85 f ,c

ρ=
0.85(21)
312 1−
[ √ 2 ( 5.67)
1 − 0.85(21) ]
ρ=0.02269

A s =ρbd A s =0.002269 (300) (550)

As =3743 mm2

c) Number of 25 mm bars:

As
Number of 25-mm
bars= As 25

25
¿
¿
¿2
Number of 25-mm π
bars= 4¿
3.743
¿

PROBLEM 2.6 (CE MAY 2009)


A reinforced concrete beam has a width of 300 mm and total depth of 600 mm. The
beam will be design to carry a factored moment of 540kN-m. Concrete strength
f ' c=28 MPa f y=248 MPa
and steel yield strength . Solve using the strength design
method.
Determine the balanced steel ratio in percent.
Determine the minimum effective depth of the beam using a steel ratio ρ
equal to 0.5 of balanced steel ratio.
c) Determine the minimum effective depth of the beam using the maximum
allowable steel ratio.
SOLUTION
Given
: f ' c=28 MPa
b=300 mm
f y=248 MPa
h=600 mm
Mu=540 kN −m

β1=0.85

a) Balanced steel ratio:


'
0.85 f β 600 0.85 (28) ( 0.85)600
c 1
ρ= ρ=
b
f y (600+ fy ) b
248 (600+248)

ρb=0.0577=5.77

ρ=0.5
b) Effective depth using

ρb
ρ=0.5(0.0577 )=0.0289

ρf 0.0289(
ω= ω=
248)y
f=' c
0.2556
28
Rn=f ' c ω (1−0.59 ω) Rn=28 (0.2556 )[1−0.59 (0.2556)]

Rn=6.0776 MPa

M u =φ M n =φ R n b d 540 x 1 0 =0.90 (8.307) (300) d


2 6 2

d=491 mm

PROBLEM 2.7
A concrete one-way slab has a total thickness of 120 mm. The slab will be reinforced
f y=275 MPa f ' c=21 MPa
with 12-mm-diameter bars with .Concrete strength .
Determine the required spacing 12 mm main bar if the total factored moment acting on
1-m width of slab is 23 kN-m width of slab is 23 kN-m. Clear concrete cover is 20 mm.

SOLUTION

Note: Slabs are practically singly reinforced because of its small depths.

. 12mm bars
d h=120 .
mm

s s
cover=20 mm

b = 1000 mm
Effective depth, d= 120 -20-1/2(12)=94 mm
Width, b = 1000 mm
94
Mu=φ 2 ¿
bd ¿
Rn 23 6
10 =0.90 R n (1000)¿
x

Rn=2.892

( √ ( √
0.85 f ' c

)
2(2.982)

)
ρ= 1− 2 Rn 0.85(21)
1− = − 1 0.85(21)
fy
0.85 f ' ρ 275 1
c

0.75 x 0.85 f ' c β1 600


ρ = =0.0284
f y (600+ f y )
ma
x

ρmin 1.4
= =0.00509
f
y

A s =ρbd A s =0.1154 (1000 )( 94)

As =1085 mm
2

Spacing of bars (for walls and slabs using unit width):

b 1000
s= N s= A
s
Ab
1000 Ab
s= Eq. 2-
As

17

12
¿
¿
¿2
π
1000 x ¿
4
1000 A b
s= As s=¿

s=100 mm

PROBLEM 2.8
A 2.8 m square column fooring has a total thickness of 47 mm. The factored moment at
f ' c=21 MPa f y=275 MPa
critical section for moment is 640 kN-m. Assume and .
Clear concrete cover is 75 mm. Determine the required number of 20 mm tension bars.

SOLUTION
Effective depth, d=470-75-1/2(20)=385 mm
Width, b =2800 mm
Mu=640 kN −m
Design strength,

Maximum and minimum requirements:

0.85 f ' c β1 600


ρ =0.75 x =0.0284
ma
x fy
(60
0+
f y)
M u max=2528 kN −m
(Procedure is not shown anymore see Problem 2.2)
1.4 bwd 2
As min = =5488 mm
fy

Singly reinforced:
385
Mu=φ 2 ¿
bd ¿
Rn
(2800)¿
6
640 x 10 =0.90 n
R

Rn=1.713 MPa

1.713
¿
2(¿ 0.85(21)¿¿)
1−¿
1−√ ¿

( √ )
0.85 f ' c 2 Rn 0.85(21)
ρ= 1− 1−
fy ¿
0.85 f ' ρ= 275
c

ρ=0.00656

A s =ρ b d A s =0.00656 (2800)(385)

As =7074 mm2> Asmin

Number of 20 mm bars:
20
¿
¿
π
4¿
As 7074
N= N=
Ab ¿

N=22.5 say 23 bars

PROBLEM 2.9
Design a rectangular beam reinforced for tension only to carry a dead load moment of
60 kN-m (including its own weight) and a live load moment of 48 kN- m. Use
f ' c=20.7 MPa f y=276 MPa.
and

SOLUTION
Required strength:

M u=1.4 M b +1.7 M L M u=1.4 (60)+1.7( 48)

Mu=165.6 kN −m

(Note: this already includes the weight of beam)

0.85 f ' c β 1600 ( )( )


ρ= ρ = 0.85 20.7 0.85 (600)
b b
f y ( 600+f y ) 276 (600+276)
ρb=0.0371

1.4
ρmi = f =0.00507
n
y

ρ=60 ρb
Try Note: this is the author’s suggestion

ρ=0.6 (0.0371)=0.02226

ρf 0.02226(276)
ω= ω=
yf ' c 20.7

ω=0.2968

Rn=f c ω ( 1−0.59 ω )
'
Rn=20.7 (0.2968 ) [ 1−0.59(0.2968 ) ]

Rn=5.068

M u=φ Rn bd 10 =0.90 ( 5.068 ) bd


2 6 2
165.6 x
2 6 3
bd =36.296 x 10 mm

Try d = 1.75 b b=228 mm say 230


mm d=399 say 30 mm

A s =ρbd A s =0.02226 (230)2(400)


2
As =2.049 mm
Summary: b = 230 mm
d = 400 mm
As =2,049 mm
2

PROBLEM 2.10
Design a singly reinforced rectangular beam for a 6-m simple span to support a
superimposed dead load of 29 kN/m and a live load of 44 kN/m. Assume normal weigth
24
γ= 3 ρmax , f ' c=34 MPa ,∧f y=345 MPa .
oncrete with m . Use
kN

SOLUTION
Weight of beam: (this is the author’s assumption)
Wb =24 x (0.3 0.6)=4.32kN / m
Assuming a 300 mm x 600 mm,

W b =1.4 W L+1.7 W L Wu=1.4 (29+4.32)+1


.7(44)
Wu=121.448 kN / m

6
¿
W L
2
¿
. =
u ¿2
8 121.448 ¿
Mu
Mu =¿

Mu=546.516 kN −m

0.05
β =0.85− =0.821
1
7 (34−30)

0.85 f ' c β 1600 ( 34 ) ( ) (600)


= = 0.85 0.821
ρb f y ( 600+f y ) ρ
b
345 (600+345)
ρb=0.04369

ρ= ρmax=0.75(0.04369) ρ=0.03277> ρmin

√ f 'c
ρmi = =0.00423 since f
>31.36 MPa
n ' c

4fy

ρf 0.03277(345)
ω= ω=
yf ' c 34

ω=0.332

Rn=f ' c ω (1−0.56 ω) Rn=34 (0.332)[1−0.59 (0.332)]

Rn=9.087 MPa

Assume d = 1.75 b (this is the author’s assumption)

75 b
1. ¿
M u = φ Rn bd
2
546.516 ¿
6
x 10 =0.90(9.087) ( b)
¿
b=279.4 mm∧d=489mm

Use b = 280 mm, d = 490 mm

Minimum beam the thickness (Section 409.6.2.1)


L f

( ) ( )
6000 345
h= +
y
hmin = 0.4+ 0.4
min 16 700
16 700
hmin=335 mm OK

A s =ρ b d A s =0.03277 (280)(490)

As =4496 mm 2

Using 32 mm bars (#100):

32
¿
¿
π
4¿
As 4496
N= N=
Ab ¿

N=5.6 say 6 bars

2
80 mm

.
. h6 -
#10

db∨25 mm
≥¿
h=490+(25)+32+20

h=554.5mm> hmin

Beam weight = 24 (0.28)(0.5545)

Beam weight = 3.73 kN/m <

4.32(OK)

PROBLEM 2.11
A propped cantilever beam shown in Figure 2.6 is made of reinforced concrete having a
width of 290 mm overall depth of 490 mm. The beam is loaded with uniform dead load
of 35 kN/m (including its own weight), and a uniform live load of 55 kN/m. Given
f ' c=24 MPa, f y=415 MPa.
Concrete cover is 60 mm from the centroid of the bars.
Determine the required tension steel area for maximum positive moment. Assume
EI=constant.

290mm

A 6m B
490 mm

2m C

Figure 2.6

SOLUTION
Given
: f ' c=24 MPa

f y=415 MPa
f yh=275 MPa

b=290 mm
D
H =490 mm
'
d =60 mm O
Lo A
L1=6 m
B 2m C
W D=35 kN / m
x
W L=55 kN / m MD
R

d=490−60=430 mm
Wu=1.4 WD +1.7 W L

W u=1.4 (35)+1.7(55)

Wu=142.5 kN / m

MA
Moment Diagram
M8

Solve for moment reactions using the three-moment equation:

M B=−142.5 (2) (1)=−285 kN−m


2 M ( L + L )+ ´
M
L +6 A 0 a´0+6 A1 b 0 =0
Mo Lo A o 1
B 1
+ L0 L1
6
¿
¿
¿3
142.5 ¿
0+2 M A (0+6)+(−285) (6)+0+¿

M A=−498.75 kN−m

A
M A=M ¿ ¿ -489.75 = R(6)- 142.5(8)

(4) R=676.875 kN

R A =W u L−R R A =142.5 (8)−676.875

RA =463.125 kN

Maximum positive moment:

V D =0 W u (2+ x )−R=0

142.5(2 + x) - 676.875 = 0
x = 2.75 m

2 2
x +¿ 2.75+¿
¿ ¿
¿2 ¿2
¿ ¿
MD =RX −Wu ¿ M D =676.875 (2.75)−142.5 ¿

MD =253.828 kN −m

φ M n max
Solve for :
ρ = 0.85 ( 24 )0.85
( )600
0.85 f ' c β1600
ρ=
f y ( 600+f y )
b b
415(600+ 415)

ρb=0.0247

ρmax =0.75 ρb ρmax =0.75(0.0247)

ρmax =0.01852

ρmax f y
ωmax 0.01852(415)
= c
ma
x
= 24
f'
ω

ωmax=0.3203

Rn max =f c ω ( 1−0.59 ω )
'
Rn max =415 (0.3203) [ 1−0.59 (0.3203) ]

430
Mn ¿
n b ¿
max=R
2
d M n max=6.235 ( 290 ) ¿
Mn max=334.316 kN−m

φ Mn max=0.90(334.316)

φMn max=300.884 kN −m

At a point of maximum positive moment:

Mu=253.828 kN −m< φ M n max


(Singly reinforced)
430
¿
Mu=φ 2 ¿
bd 6
Rn 253.828 10 =.90 R n (290) ¿
x
Rn=5.26 MPa

√ ]
0.85 f ' Rn
1−
ρ= c 1− 0.85 f 'c
fy

0.85(24 )
[
[ √ ]
ρ 415 1
2( 5.26)
= − 1− 0.85( 24)

ρ=0.01495

A s =ρ b d A s =0.01495 (290)(430)
2
As =1,864 mm
ANALYSIS OF RECTANGULAR BEAMS
f =f ¿
WHERE STEEL YIELDS ( S Y

PROBLEM 2.12(CE MAY 1999)


A reinforced concrete rectangular beam with b = 400 mm and d= 720 mm is reinforced
for tension only with 6-25 mm diameter bars. If f c=21 MPa and
'
f y=400 MPa, determine the following
:
Rn
a) The coefficient of resistance of the beam.
b) The ultimate moment capacity of the beam.

SOLUTION
0.85 f 'c β1
ρ= 0.85 ( 21 ) ( 0.85 )( 600)
ρb =
f y (600+ fy )
b
400(600+ 400)

ρb=0.02276

25
¿
¿ π
A =6 x ¿
s
4

2945
ρ= As ρ = =0.01023< ρ ( steel yields )
b
bd 400(720)

ρf 0.01023(400)
ω= ω= =0.195
yf ' c 21
R n =f 'c ω(1−0.56 ω) Rn=21 (0.195)[ 1−0.59 (0.195)]

Rn=3.62 MPa
Answer
720
¿
Mu=φ 2 ¿
bd
Rn Mu=0.90 (3.62) ( 400) ¿

Mu=675.67 kN −m
Answer

PROBLEM 2.13

A rectangular beam reinforced for tension only has b= 300 m, d = 490 mm. The tension
steel area provided is 4,500 sq. mm. Determine the ultimate moment capcity of the
f ' c=27 MPa f =275 MPa.
beam in kN-m. Assume , y

SOLUTION
0.85 f ' c β1600 ( 27) ( ) (600)
= = 0.85 0.85
ρ b
f y ( 600+f y ) ρ
b
275 (600+275)

ρb=0.02276

As
ρ=
bd 4,500
ρ= 300(490)
ρf 0.0361(275)
ω= ω=
yf ' c 27

ω=0.3118

R n=f 'c ω(1−0.59 ω) Rn=27 (0.3118 )[ 1−0.59 (0.3118 )]

Rn=6.87 MPa
490
¿
Mu=φ 2 ¿
bd
Rn Mu=0.90 (6.87) (300) ¿
Mu=445.3 kN −m

PROBLEM 2.14

A s =3−25 mm, f ' c=34.2


A rectangular beam has b = 300 mm, d = 500 mm,
MPa ,
f y=414 MPa Mu.
grade 60 reinforcement ( Calculate the design moment
¿.

SOLUTION
0.05
β =0.85− (34.2−30)=0.82
1
7

0.85 f ' c β1600


ρ=
0.85 ( 34.2) ( 0.82) (600)
b
f y ( 600+f y ) ρb=
414(600+ 414)

ρb=0.03407
25
¿
¿π
A= ¿
s
4

As
ρ=
bd 1473
ρ = 300(500)

ρ=0.00982< ρb Steel yields

Check if the beam satisfies the minimum requirement:


ρmin = √ f ' c =0.00353 OK
4f
y

ρf 0.00982(414)
ω= ω=
yf ' c 34.2

Rn=f c ω ( 1−0.59 ω ) Rn=34.2 ( 0.1188 ) [ 1−0.59 ( 0.1188 ) ]


'

Rn=3.779 MPa

500
¿
¿
Mu=φ 2 =0.90(3.779 ) (300)
bd u
Rn M ¿
Mu=255.11 kN −m

PROBLEM 2.15
A 130-mm-thick-one-way slab is reinforced with 12-mm-diameter tension bars spaced
f ' c=21
at 110 on centers. Concrete cover is 20 mm, concrete strength MPa and
f y=275 MPa 3
steel yield strength . Unit weight of concrete is 23.5 kN/ m .
What is the ultimate moment capacity of the slab?
If the slab is simply supported over a span of 4 m, what safe uniform live load
pressure can the slab carry?

SOLUTION
a) Consider 1 m width of slab, b = 1000
db
mm Effective depth: d = h – cover- 1/2
d = 130-20-1/2(12)=104 mm

0.85 f ' c β 1 600 ( 21) ( ) (600)


= = 0.85 0.85
ρb f y (600+f y ) ρb 275( 600+275)

ρb=0.0378

b
As =Ab x N As =Ab x
s

12
¿
¿π
A= ¿
s
4
As =1028 mm
2

As 1028
ρ= ρ=
bd 1000( 104)

ρ=0.00989

Check if the beam satisfies the minimum steel requirement on flexures:


ρmi 1.4
= =0.00509 OK
n f
y

ρf 0.00989( 275)
ω=
fy ' c =
ω
21
ω=0.129

Rn=f ' c ω (1−0.59 ω )Rn =21(0.129 ) ( 1−0.59


(0.129) ]

Rn=2.511 MPa

104
Mu=φ bd
2
¿
Rn ¿
Mu=0.90 2.511) (1000)¿
(

Mu=24.443 kN −m

4
¿
¿
¿2
b) W2 ¿
WuL u
Mu= 24.443=¿
8
Wu=12.222 kN / m

ρ D=γc
Dead load pressure, x thickness of
concrete. ρD=23.5 x 0.13=3.055
Dead load pressure,
kPa
Wu=1.4 W L +1.7 W L Wu=1.4 ( ρD b )+1.7 ( ρ L b )

12.222=1.4 (3.055 x 1)+1.7( ρL x 1)

ρ L=4.673 kPa

PROBLEM 2.16

A rectangular beam with b = 250 mm and d = 460 m is reinforced for tension only with
3-25 mm bars. The beam is simply supported over a span of 6 m and carries a uniform
dead load of 680 N/m including its own weight. Calculate the uniform live load that the
f y=276.5 f ' c=20.7 MPa
beam can carry. Assume and .
MPa
SOLUTION
25
¿
¿ π
A =3 x ¿
s
4

0.85 f ' c β1600 ( 20.7) ( ) (600)


= = 0.85 0.85
ρb f y ( 600+f y ) ρb 276.5 (600+276.5)

ρb=0.03703

As 1.473
ρ=
bd = 250 (460)
ρ

steel yields
ρ=0.01281< ρb ¿ )
Check if the beam satisfies the minimum steel requirement on flexure:

ρmi 1.4
= =0.00506 OK
n f
y

ρf 0.01281(276.5)
ω=
fy ' c =
ω
20.7

ω=0.171

Rn=f 'c ω (1−0.59 ω )Rn =27 (0.171)[1−0.59 (0.171)]

Rn=3.183 MPa

460
¿
¿
Mu=φ 2 u=0.90(3.183 ) (250)
bd
Rn M ¿
Mu=151.56 kN −m

6
¿
¿
¿2
Wu2¿
W L
u
M = 151.56=¿
umax 8

Wu=33.68 kN /

m Wu=1.4 W DL +1.7 W ¿ 33.68=1.4 (0.68)+1.7 W ¿


W ¿=19.25 kN −m

PROBLEM 2.17 (CE JANUARY 2008)


A reinforced concrete rectangular beam has a width of 300 mm and an effective depth
of 55 mm. The beam is reinforced with six 25-mm-diameter tension bars. Steel yield
fy
is 415 MPa and concrete strength f ' is 28 MPa.
c
What is the balanced steel ratio?
What is the maximum steel area for singly reinforced?
What is the nominal moment capacity of the beam?

SOLUTION
a) Balanced steel ratio:

0.85 f ' c β1600 ( )( )


ρ= ρ = 0.85 28 0.85 600
b b
f y ( 600 +fy ) 415(600+ 415)

β1=0.85 ρb=0.028816
ρb=2.88

b) Maximum steel area

A s max =ρmax bd A s max=( 0.75 ρb )


bd
A s max =(0.75 x 0.028816) (300 )( 5)

As max =3,242 mm2

c) Nominal moment
capacity Using 6-25 mm
bars:
25
¿
¿π
A= ¿
s
4

As 2,945
ρ=
bd = 300 (500)
ρ

ρ=0.01963< ρb (tensio steel yields)

ρf 0.01963(415)
ω= ω= =0.291
yf ' c 28

Rn=f c ω ( 1+0.59 ω ) R n=28 ( 0.291 ) (1−0.59 x 0.291)


'

Rn=6.7494 MPa

500
¿
2
¿
Mn=Rn bd Mn=6.7494 (300)¿

Mn=506.2 kN−m
PROBLEM 2.18
A 350 mm x 500 mm rectangular is reinforced for tension only with 5-28 mm bars. The
beam has an effective depth of 446 mm. The beam carries a uniform dead load of 4.5
kN/m (including its own weight), a uniform live load of 3 kN/m, and concentrated dead
f y=414 MPa , f ' c=34.5
load of P and 2P as shown in Figure 2.7. Assume .
MPa
Calculate the following:
The ultimate moment capacity of the section in kN-m, and
The maximum value of P in kN.

2P P

2m 2m 2m

Figure 2.7

SOLUTION
0.05
β =0.85− (34.5−30)=0.818
1
7

0.85 f ' c β 1 600 ( 34.5) ( ) (600)


= = 0.85 0.818
ρb f y (600+f y ) ρb 414( 600+414)
ρb=0.03428

28
¿
¿π
A= ¿
s
4

As 3079
ρ=
bd =300 (446)
ρ

ρ=0.01972< ρb Steel yields

Check if the beam satisfies the minimum requirement:

√ f 'c
m ∈¿= =0.00355
4fy
ρ¿

ρf 0.01972(414)
ω= ω=
yf ' c 34.5

ω=0.2367

Rn=f ' c ω (1−0.59 ω )Rn =34.5 (0.2367)[ 1−0.59 (0.2367)]

Rn=7.025 MPa
b
M
u

=
φ
R
n
Mu=440.18 kN −m

1.4(2P)
1.4P

W u=1.4 (4.5)+1.7 (3)=11.4 kN / m


A

Ra B C
D

2m 2m
2m

Figure 2.8 – Beam with factored loads

For the given loads, the maximum moment can occur at B or C:

Mc=1.4 P (2)+11.4 (2)(1)


At point C:
Mc=Mu
Set 440.18 = 1.4P(2) + 11.4(2)(1)
P=149 kN
RA ¿
At point B: (First solve for

∑ M =0 4 Rc A +1.4 P (2)=2.8 P (2)+11.4 (6)(1)

RA =17.1+ 0.7 P

MB ¿ M = (17.1+0.7 P )−11.4 (2 ) (1)


B

∑¿
M B=M u 440.18=(17.1+0.7 P) (2 )−11.4 (2)(1)
Set
P=306.27 kN

Mu
Thus the maximum value of P such that will not exceed 440.18 kN-m is 149 kN.

ANALYSIS OF RECTANGULAR BEAMS


f S≠ f Y
WHERE STEEL DOES NOT YIELDS (
)

PROBLEM 2.19

As =6−32mm , f ' c=27.6 MPa ,


A rectangular beam has b = 300 mm, d = 500 mm,
f y=414 MPa
grade 60 reinforcement ( Calculate the ultimate moment capacity of
¿.
the beam.

SOLUTION
0.85 f ' c β1600 ( 27.6) ( ) (600)
= = 0.85 0.85
ρb f y ( 600+f y ) ρb 414 (600+414 )
ρb=0.0285

32
¿
¿π
A= ¿
s
4

As 4825
ρ=
bd = 300 (500)
ρ

ρ=0.03217> ρb Steel does not yield

b=.300 a
0.85 b
c=0.8
5
a
d=5
00

500-a/2

=482
From Eq. 2-1
5
d−c8
f 600 500−c
= =600
s T=
f c
s
c

∑ FH =0
T =C A s f s=0.85 f ' c ab, a=β 1 c=0.85 c

500 −c
(4825)600 =0.85 (27.6) (0.85 c )(300)
c

2
c =484 c−241,964=0
c=306.2 mm
d−c
f 600
= f s =600 500−306
306
s
c

f s=379.65 MPa

a=β 1 c=0.85 (306.2)

a=260.3 mm

a
φM = φ A f d −
n s s ( 2)
260.3
φMn =0.90 ( 4825) (379.65)(500− )
2

φ Mn=609.8 kN −m
PROBLEM 2.20

A rectangular beam reinforced for tension only has b=300 mm, d = 490 mm. The
f y=415 MPa.
tension steel area provided is 7-25 mm diameter bars with
f ' c=21 MPa
. Calculate the ultimate moment capacity of the beam.

SOLUTION
0.85 f ' c β 1 600 ( 21) ( ) (600)
= = 0.85 0.85
ρb f y (600+f y ) ρb 415( 600+415)

ρb=0.02161

25
¿
¿π
A= ¿
s
4

As 3436
ρ= ρ=
bd 300 (490)

ρ=0.02337> ρb Steel does not yield


0.85 a
b=300
b
c=0.8
a 5
d=4
90

490-a/2
=343
6

T=
From Eq.2-18:
d−c
f 600
= =600 490−c
f s
c
s
c

∑ FH =0

T =C A s f s=0.85 f ' c ab, a=β 1 c=0.85 c

490−c
(3436)600 =0.85 (221) (0.85 c )(300)
c

c=296.24 mm

d−c
f 600
= f s =600 490−296.24
296.24
s
c

f s=392.43 MPa< f y

a=β1 c=0.85 (392.43)

a=251.81mm
a
2
n s s (d− 2 )
251.81
φ Mn =0.90 (3436) (392.43)(490− )
2
φMn=441.86 kN−m

ANALYSIS & DESIGN OF SINGLY REINFORCED


NON-RECTANGULAR BEAMS
PROBLEM 2.21

Compute the ultimate moment capacity of the beam shown in Figure 2.9. Assume
f y=345 MPa f ' c=21 MPa
and .

125125 125

125
700m
m

4-32mm

75
375mm

Figure 2.9
SOLUTION
Note: This is not a rectangular beam. Some formulas derived above (such as
ρ , ρb Rn
, ) may not be applicable. The moment can be computed using the
assumptions in the Code and the conditions of equilibrium.

32
¿
¿π
A= ¿
s
4

As =3217 mm
2

As
Solve for the balanced to determine whether the given steel yield or not.

Cb =: 125125125
From Eq. 2-11
12
5

cb =
600 d
C=
600(625) a
600+ fy b 600+345
625m
m

4-
32mm
375mm
Cb =396.825 mm

a=β 1 c a=0.85(396.825)

a=337.3 mm

A c =337.3 ( 375 )−125 ( 125 ) =110,863 mm


2

T =C Asb f y=0.8 f ' c Ac

A sb (345)=0.85 (21)(110,863)

Asb=5,736 mm 2

A s provided Asb
Since < , tension steel yields.

C=T 0.85 f c (ab−125 )= As f


' 2

0.85 ( 21 ) ( a x 375−125 )=3,217(345)


2

a=207.5 mm
121521525 b=375
a
1
2
5

C a 625mm
d-a/2
4-
N3 .
2A
m
375mmm
II

I
a 125
( )
M n=M n 1−M n 2 M n=C 1 d−2 −C2 (d− 2 )

207.5
M =0.85 (21) (207.5 ) (375)(625− )
n
2

Mn=567.03 kN −m

φMn=0.90( 567.03)

φMn=510.33 kN −m

PROBLEM 2.22

Compute the ultimate moment capacity of the beam shown in Figure 2.10. Assume
f y=345 MPa f ' c=21 MPa
and .

a
450m

c
37
5

450m
450m
m

m
3-22mm

375
x d-(2/3)a

d-
75
375mm
3-

22mm
Figure 2.10 75
T
375mm
SOLUTION
22
¿
¿π
A= ¿
s
4
As =1,140 mm
2

As :
Solve for

600 d
Cb =
600+f y

600(375)
Cb=
600+ 345

Cb =238 mm

ab =β 1 C b ab =0.85(238)

ab =202.4 mm

x 375 5
a = 450 x =6 a

x=168.7 mm

1 5 5 2
A =1/ 2( x )(a) A= x a xa= a
c c
2 6 a
Ac=17,066 mm
2

T =CC Asb f y=0.85 f ' c Ac

A sb (345)=0.85 (21)(17,066)

Asb=883 mm 2< As

As provided > Asb f s <f y ¿


Since , tension steel does not yield (
solve for c:

CC =T f ' c Ac= A s f s
0.85

d−c 5 2 d −c
f =600 0.85(21) a =1140 x 600
s
c 12 c
85 c
a=β1 c 0. ¿
¿
7.437 ¿

c=250.92 mm

a=β1 c a=0.85 (250.92)=213.3 mm

2 2
( ) ( )
'
M n=C c x d− 3 a M n=0.85 f c Ac d− 3 a
29
213. ¿
¿
5
M =0.85 (21) ¿
n
12

Mn=78.77 kN −m

φ Mn=0.90 x 78.77=70.89 kN −m

PROBLEM 2.23
f ' c=28 MPa f y=345 MPa
A hallow beam is shown in Figure 2.11. Assume and .
Mu=800 kN
a) Calculate the required tension steel area when .
−m
What is the balanced moment capacity of the beam?
What is the maximum steel area under singly reinforced condition?
What is the maximum design moment strength under singly reinforced condition?
Mu=1200 kN
Calculate the required tension steel area when .
−m

500 mm
125 250 125
150
800 mm
500
150

75 mm
Figure 2.11- Hallow beam

SOLUTION
To guide us whether “a: will exceed 150 mm or not, let us solve the design
moment when a=150 mm.

d = 800 – 75 = 725 mm

( )
φ M n=φ C C d−a
2
(
φM n=0.90 x 0.85(28 ) (150) 725−150
2
)
φMn=1044.225 kN −m

Mu=800 kN −m
a)
Mu=800 kN −m <1044.25 kN−m, a<150 mm.
Since the required

Assuming tension steel yields:


a
M =φM M =φ C ( d− )
u n u c
2
Mu=φ 0.85 f ' c
a
ab (d− )
2

800 x 10 =0.90 x 0.85 ( 28 ) a ( 500 ) (725−0.5 a)


6

a=111.6 mm<150 mm

Check is steel yields:


d−c a
f =600 where c = =131.3 mm
s
c β1
725−131.2
f =600 =2,712 MPa>f
steel yields
s 131.3 y
T =C As f y=0.85 f ' c ab

A s (345)=0.85(28 ) (111.6 )(500)

As =3,850 mm 2

b) Balanced condition (See Figure 2.12)


600 d 600(725)
C= C= =460.32mm
b b
600+f y 600+345

a=β1 C b a=0.85 (460.32)=391.3 mm

z=a−150=241.27 mm
2 1
A =500 ( 150 )=75,000 mm y =725− =650 mm
1 1
2 (150)

2 1
A =125 (241.27)=30,159 mm y =725−150− =454.37
1 2
2 (241.27)

M bn=C1 y1+ 2C2 y2


A1 y1 +2 A2 y2
c
M bn =0.85 f ' ¿ )

M bn=0.85 (28) [75,000 x 650+ 2 x 30,159 x 454.37 ]

M bn=1812.52 kN−m

φ Mn=0.90 x 1812.52=1631.3 kN−m

5
1 02 1
2 0 1
150

5
m
2 0 22
za
7
2
5

m
5 5
Figure
2.12 T
As max
,
c) Maximum steel area
'
T =C1 +C 2 Asb f y=0.85 f c( A1+2 A2 )

A sb (345)=0.85 (28)(75,00+ 2 x 30,159)

Asb=9,335 mm
2

As max =0.75 Asb As max =0.75 (9,335)

As max =7,001 mm
2

M u max :
d) Maximum moment ,
Refer to Figure 2.12:
C1 +C 2=T 0.85 (28) [ 75,000+2 A 2 ]=7,001(245)

A2=13,244 mm 2

A2=125 z 13,244=125 z

z=105.95 mm

1501
y =725− =522.03 mm
2
2 (105.95)

M n max=C 1 + y1+ 2C 2 y 2

Mn max=0.85 f c (A1 y1 +2 A2 y2)


'

M n max=0.85(28 ) [ 75,00 x 650+2 x 13,244 x 522.03 ]

M n max=1489.34 kN−m

φMn max=0.90 x 1189.34=1340.4 kN −m

e) Mu=1200 kN −m< φ M n max (Singly reinforced )

Refer to Figure 2.12

A1=75,000 mm y1=650 mm
2

A2=125 z y2 =575−0.5 z

Mu=φMn
6 '
1200 x 10 =0.90 x 0.85 f c ( A1 y1 +2 A2 y 2)

1200 x 10 =0.90 x 0.85 ( 28 ) [75,000 ( 650 ) +2 ( 125 z ) ( 575−0.5 z ) ]


6

z=53.04 mm

A c = A1+ A2 Ac=75,000+2 x 125 (53.04 )

Ac=88,259.2 mm
2

T =C As f y=0.85 f ' c Ac

A s (345)=0.85(28 )(88,259.2)

As =6,089 mm
2

BEAM DEFLECTION

PROBLEM PROBLEM 2.24


A reinforced concrete beam is 350 mm wide and 600 mm deep. The beam is simply
supported over a span of 8 m and carries a uniform dead load of 11 kN/m including its
own weight and a uniform live load of 15 kN/m. The beam is reinforced tension bars of
f ' c=20.7 MPa, f y=344.8 MPa, f r=2.832 MPA .
530 mm. Modulus of elasticity of
Ec=21,650 MPa Es =200 GPa.
concrete and
Calculate the maximum instantaneous deflection due to service loads.
Calculate the deflection for the same loads after five years assuming that 40% of
the live load is sustained.
SOLUTIO
N b = 350 b = 350 mm

h = 600 mm
mm
c

d = 530
N.A.

mm
6 – 25 mm d-c
Ø
Figure 2.13

I e:
Effective moment of inertia, Eq. 2-19

( [ ( )]
3
Mc M cr 3
I e=
I g + 1− M
r
M a Icr g

) ≤I

I g=moment of inertia of gross section

600
¿
¿
¿3
3503 ¿
bh
I g= I g =¿
12

f rI g
M = where y =1/ 2(600)=300 mm
cr t
yt

10
600 x ¿
¿
¿6
2.832 ¿
M cr =¿
Ma=Maximum monet ∈beam

2
wL
Ma =
w=wD + wL=11 +15=26 kN / m
8

8
¿
¿
¿2
26 ¿
Ma=¿

I cr=¿
Moment of inertia of cracked section with steel transformed to concrete
From Figure 2.13:

Es
Modular n= =9.238
Ec
ratio,

25
¿
¿ π
n A =9.328 x 6 x ¿
s
4

Solve for c:
Moment of area above N.A. = Moment of area below N.A.
350 x c x c/2 = 27,208(350-c)
c = 219.7 mm
d−c
¿
¿ 3
bc
I cr=I NA= +n As ¿
3

219.7
¿
¿
¿3
350 ¿
I cr=¿

I cr=3,857 x 106 mm 3
( [ ( )]
3
Mc M cr 3
I e=
I g + 1− M
r cr
M a
a

) I

[(
3 3

( ) )
59.472 6 59.472 6
I e= x 600 x 10 + 1− x 3,857 x 10
208 208

]
I e=3,914 x 106 mm4 < I g (OK )

a) Instantaneous Deflection:
2 ( 26 ) (8000)
4 4
5 wL
δ= δ= 6
384 Ec Ie 384 (21,650)(3,914 x 10 )

δ =16.36 mm
b) Long-term Deflection
Since only 40% of the live load was sustained:
w = 11 + 0.4(15) = 17 kN/m
4
5 wL
δ=
Instantaneous deflection 384 Ec I e

5 ( 17 ) (8) (1000)
4 4

δ= 6
384 (21,650)(3,914 x 10 )

δ =10.7 mm

Note: Since deflections are directly proportional to the load, the instantaneous
deflection due to sustained load can be found by ratio and proportion using the result in
Part”a”.
δ1 16.36
=
17 26
δ 1=10.7
m

δ + δ1
Long-term deflection =

ξ
λ = 1+50 ρ '

ξ=2 for 5 years ∨more


'
ρ =0 since there is no compression reinforcement

2
λ= =2
1+50(0)
Long-term deflection = 16.36 + 2(10.7)
Long-term deflection = 37.76 mm

PROBLEM 2.25 (CE NOVEMBER 2002)


The continuous reinforced concrete beam shown in Figure 2.14 is subjected to a
uniform service dead load of 16 k/m and a service live load of 32 kN/m,resulting in the
bending moment diagram shown. Twenty percent of the live load will be sustained in
nature, while 80% will be applied only intermittently. The concrete strength
f c=17.2 MPa.
The modulus of elasticity of concrete is given by the expression
Ec=4700 √(f ' c) and the modulus of rapture is given by the expression
f r =0.7 √( f c)
'
. Determine the following:
The effective moment of inertia at the supports (maximum negative moment).
The effective moment of inertia for the continuous member.
The additional deflection (in addition to the initial deflection) after 5 years, under the
sustained loading if the instantaneous deflection due to the combined service dead
and live load is 5 mm.

7.6
- 3 m
5 2 mmø 3-32 5- 2
mmø 3 mmø
145 kN-m

202 kN-m
202 kN-m
y
560 mm
y
Gross Section Cracked
Section I=0.0715 I=0.00573
y=310 mAmT SUPPOy=R1T5S9 mm
1900 mm
620 mm

y y
560 mm
n As

Gros Crack
s
AT MIDSPAN ed
Sec Sec
t ti
ion on
I=0 I=0.
. 0
01 057
3 3
8 y=10
y= 7 mm
1
94
mm

Figure 2.14

SOLUTION
Ec =4700 √ f 'c =4700 √17.2=19,492 MPA

f r=0.70 √ f ' c =0.7 √ 17.2=2.903 MPa

a) Effective moment of inertia at the supports


Mu=202 kN −m
Maximum moment,
Y t =310 mm
Distance from NA of gross section to extreme tension fiber,
I g=0.00715 m
4
Moment of inertia of gross section,
I g=0.00573 m
4
Moment of inertia of cracked section,
M cr f I
r
M cr 2.903 (0.00715 x 1000
4
)
= g 10
yt =

M cr =66.959 kN−m

( [ ( )]
3
Mc M cr 3
I e=
I g + 1− M
r cr
M a
a

) I

( ) x 0.00715+[1−(
3

)
3
66.959 66.959
I e= x 0.00573
202 202

]
I e=0.0057817 m4

b) Effective moment of inertia for the continuous member

Ie
¿
¿
I
¿
I e=¿
At maximum negative moment (at support)

I e=0.0057817 m4

Ie
Solving for at maximum positive moment (at midspan)
I g=0.0138 m
4

Y t =620−194=246 mm (bottom fibers∈tension )

I cr=0.00513 m
4

4
f rI 2.903(0.00715 x 1000
M cr
g M cr )
= Yt = 310

M cr =66.959 kN−m

( [ ( )]
3
Mc M cr 3
I e=
I g + 1− M
r cr
M a
a

) I

Ie
¿
¿
I
¿
I e=¿

0.0057817+ 0.007932 4
I e= =0.006857 m
2
λ
c) Additional long term deflection= long term deflection x

ξ
λ = 1+50 ρ '

ρ =0 (since there is no compression reinforcement at midspan )


'

ξ=2( after 5 years )

2
λ= =2
1+0

Solving for the instantaneous deflection under sustained loading:

Instantaneous deflection = 5mm (given)


Instantaneous loading = 16 kN/m + 32 kN/m
Instantaneous loading = 48 kN/m

Sustained loading = 16 + 20%(32)


Sustained loading = 22.4 kN/m

Sine deflection is directly proportional to the load:

δ1
5
22.4 = 48

δ 1=2.33 m
λ
Additional long term deflection = 2.333 x
=2.333 x 2
Additional long term deflection = 4.67 mm

ONE-WAY SLAB

Reinforced concrete design slabs are large flat plates that are supported at its sides by
reinforced concrete beams, walls, columns, steel beams, or by the ground. If a slab is
supported on two opposite sides only, they are referred to a one-way slabs since the
bending occurs in one direction only. If the slab is supported on all four sides, it is called
two-way slab since the bending occurs in both direction.

If a rectangular slab is supported in all four sides but the long is two or more times the
short side, the slab will, for all practical purposes, act as one way slab, with bending
occurring in the short direction.

b = 1m

h
Figure 2.15: One-way slab on simple support
A one-way slab is considered as a wide, swallow, rectangular beam. The reinforcing
steel is usually spaced uniformly over its width. One way-way slabs are analyzed by
considering one-meter strip, which is assumed independent of the adjacent strips. This
method of analysis is somewhat conservative because we neglect the lateral restraint
provided by the adjacent strips.

MAXIMUM SPACING OF REINFORCEMENT


According to Section 407.7.5, the flexural reinforcement shall not be spaced farther
apart than 3 times the slab thickness, nor 450 mm.

ρT
SHRINKAGE AND TEMPERATURE REINFORCEMENT,

Concrete shrinks as it hardens. In addition, temperature changes occur that causes


expansion and construction of concrete. In this effect, the code (407.13) requires that
one-way slab, where flexural reinforcement extends in one direction only, should be
reinforced for shrinkage and temperature stresses perpendicular to flexural
reinforcement. According to Section 407.132.2.1, the area of shrinkage reinforcement
shall provide at least the following ratios of gross concrete area bh, (where h is the slab
thickness) but not less than 0.0014.

a) Where Grades 230 & 275 deformed bars are used.........................0.0020


b) Where Grade 415 deformed bars or welded wire
fabric (plain or deformed ) are used.....................................................0.0018
f y> 415 MPa
c) Where reinforcement with measured at
yield strain of 0.35% are used………………………………………….
0.0018 x 415
fy

Shrinkage and temperature reinforcement may not be spaced not farther apart than 5
times the slab thickness, nor 450 mm (Section 407.13.2.2).

STEPS IN THE DESIGN OF ONE-WAY SLABS (FLEXURE)


I. Identify the uniform floor pressure (Pa) to be carried by the slab. This load may
consist of:
1) Live load pressure
2) Dead load pressure
3) Ceiling load and other attachments below the slab
II. Determine the minimum slab thickness “h” from Table 2.1. If necessary adjust
this value depending on your judgment.
III. Compute the weight of slab
γ xh
(Pa) Weight = conc
Mu ¿
IV. Calculate the factored moment ( to be carried by the
slab. Wu=Factored pressure x
Uniform load,
1m
.
V. Compute the effective depth, d:
d=h-covering (usually 20 mm)-1/2 (main bar diameter)
ρ
VI. Compute the required steel ratio :
M u =φ Rn bd where b=1000 mm
2
Rn
Solve for from
√ ]
0.85 f ' 2 Rn
1−
ρ= c 1− 0.85 f 'c
fy

Solve for

If
ρ
[
ρmin

is less than
ρmax
and greater than
ρmin
, use
ρ

ρ ρmax
If is greater than , increase the depth of slab to ensure
ductile failure
ρ ρmin , use ρ=ρmin
If is less than
VII. Compute the required main bar spacing.
A s =ρ bd=ρ (1000) d≥ ρt bh

A ¯¿
x 1000
Spacing As
, S1=¿
Use the smallest of the following for the main bar spacing:
S1
a)
b) 3 xh
c) 450 mm

VIII. Temperature bars: See Page 81 for the required steel ratio,

ρt
Ast =ρt bh

A ¯¿
x 1000
As
S2=¿

Use the smallest of the following for temperature bar spacing:

S2
a)

b) 5 xh

c) 450 mm

ILLUSTRATIVE PROBLEMS

Problem 2.36
Design a one-way slab having a simple span 3 m. The slab is to carry a uniform live
load of 7,500 Pa. Assume and for main
and temperature bars. The slab is not exposed to earth or weather. Use unit weight of
concrete .

SOLUTION
Consider 1 m strip of slab, b= 1000 m
wL=7.5 KPa x 1 m=7.5 kN /m
Uniform live load,

Minimum slab thickness from Table 2.1:

L f

(0.4+ 700 ) ( )
3000 276
y
=
hmin +
0.4
hmin= 20 700
20

hmin=119 mm (use 120 mm)

Effective depth:

10 mm
temp. Bba=rs1000
mm
h =d 120 mm

102mm main
bars Cover + /2
d = 120-20 mm (covering)-1/2 bar diameter (12mm)
d=94 mm

Weight of slab:
W s =γconc xb x h W s =23.5(1)(0.12)
Ws 2.82 kN / m

Factored floor pressure load:

Wu=1.4 ws + 1.7 w L Wu=1.4 (2.82)+1.7 (7.5)

Wu=16.698 kN / m

2 2
WuL 16.698(3)
M = M=
u u
8 2

Mu=18.785 kN −m

M u=φ Rn b d 18.785 x 10 =0.90 R n ( 1000 ) (94)


2 6 2

Rn=2.362 MPa

0.85 f ' R

[√ ] [ √ ]
ρ= 1− 1−
f c u 0.85 f ( 27.6 )
0.85276 2 ( 2.362 )
0.85
' ρ= 1− 1−(27.6)
y c

ρ=0.009039
ρmin ρmax
Check for and :

ρmi 1.4
= =0.00507 OK
n f
y

0.75 0.85 f 'c β1 600


ρ =
ρ 0.75 0.85 ( 27.6) ( 0.85) 600
=
max
f y (600+ f
ma
x 276 (600+276)
y)

ρmax =0.037> 0.009309(OK )

A s =ρbd A s=0.009039 (1000)


(94)
A s =850 mm
2
per meter width of slab

Using 12-mm main bars:

12
¿
¿
¿2
π
Spacing s 4¿
= A ¯¿
x 1000
s=¿
As
¿

s=138 mm say 135 mm

Maximum spacing required by the


Code: 3 (h)=3 (120)=360 mmOK
a)
450 mm
b)
Thus, use 12 mm main bars at 135 mm o.c.

Temperature bars: (Grade 275)

A t =0.002 bh A t=0.002(1000) (120)

At =240 mm
2

10
¿
¿
¿2
π
Spacing 4¿
= A ¯¿
x 1000
s=¿
As
¿

s=327 mm say 325 mm

Maximum spacing required by the


Code: 5 h=5 (120)=600 mm
a)
b) 450 mm OK

Thus, use 10 mm temperature bars at 325 mm o.c.

10 mm main
120 mm

tempe1ra2tmur bars
me bars @ 325 mm o.c.
mm@o3.2c5. mm
20
L= 3m
PROBLEM 2.27
Design a one-way slab to carry a service live load of 4000 Pa. The slab has a length of
4m with both ends continuous. Assume and for main

bars and for temperature bars. Steel cover is 20 mm. Unit weight of

concrete is 23.5 kN/ .

SOLUTION
Consider 1 m strip, b = 1000 mm
wL =4 kpa x 1 m=4 kN / m
Uniform live load,

Minimum slab thickness from Table 2.1:


L
h = 4000
h =
min min
28 28

hmin=143 mm(use 150 mm)

Weight of beam (DL):


w D =γ conc x b x h w D =23.5 (1)(0.15)

wD =3.525 kPa

w u=1.4 w D + 1.4 w L w u=1.4 (3.525)+1.7( 4)

wu=11.735 kN / m

Maximum factored moment, Section 408.4 (See Page 29)


LL < 3 DL

Column Column Column

Spandr
el
Bea
m

Shear

Momen
t

Effective depth, d = 1.50 – 20 – 1/2 (12)


Effective depth, d = 124 mm

At midspan:
Mw1
uu 1
.
L
7
2
n
3
5
1(
4
6
)
2
M
1
u
6
=
Mu=11.735 kN −m
124
¿
¿
2 6
Mu=φ bd 11.735 x 10 =0.90 Rn(1000)¿
Rn

ρ=
0.85
f f'
y
1−
√ Rn=0.848 MPa

] ]
1− 2 Rn


0.85 ( 21 ) 2 ( 0.848 )
c 0.85 f ρ= 1 − 0.85(21)
'

[
c

[
1−
415

ρ=0.0021

ρmi 1.4
= =0.00337> 0.0021
n f
y

ρ= ρmin=0.00337
Use

A s =ρbd A s=0.00337 (1000)(124)

As =418 mm
2

π 2
Asb (12)
4
Spacing, s = x 1000 s= x 1000
As 418
s=271 say 270 mm

Maximum spacing required by the


Code: 3 h=3 (150)=450 mm
a)
b) 450 mm

Thus, use 12 mm bottom bars at 270 mm o.c. at midspan

At support:
2 2
w L 11.735(4)
M u= 10 M =
u n
u
10

Mu=18.776 kN −m

M u=φ Rn bd 18.776 x 10 =0.90 Rn ( 1000 ) (124)


2 6 2

Rn=1.357 MPa
[ ]0.85 ( 21 ) 2(1.357)
]
1−2 Rn
ρ= 1−
0.85 f ' 1−

√ √
ρ= c 1−

fy 0.85 f 415 0.85( 21)

'c
[
ρmax =0.0034> ρmin

0.85 f 'c β1 600


ρ =0.75
ρ 0.85 ( 21) ( 0.85) 600
ma ma =0.75
x f y (600+ f y) x 415( 600+415)
ρmax =0.0162>0.0034

ρ=0.034
Use

A s =ρbd A s =0.0034 (1000)(124)

As =422 mm
2

π 2
A sb (12)
4
Spacing, s= x 1000 s= x 1000
As 422

Spacing =268 say 265 mm

Thus, use 12 mm top bars @ 265 mm o.c. at support

ρt =0.002 ¿
Temperature bars (10 mm): (

A t =0.002 bh A t=0.002(1000)(150)

At =300 mm
2
10
¿
¿
¿2
Spacing, s π
= 4¿
Asb
x 1000 s=¿
As

s=261 say 260 mm

Maximum spacing required by the


Code: 5 h=5 (150)=750 mm
a)
b) 450 mm

Thus, use 10 mm temperature bars @ 260 mm o.c.


150 mm

10 mm temperature
bars @ 260 mm o.c.

12 mm main bars
L/4 @ L/4
265 m m
L /2
o.c.
PROBLEM 2.28

A one-way slab having a simple span of 3 m is 160 mm thick. The slab is reinforced
with 12 mm tension bars spaced at 140 mm o.c. Steel covering is 20
mm. Calculate the uniform live load pressure that a slab can carry. Use

. Unit weight of concrete is 23.5 kN/ .

SOLUTION

Consider 1 m strip of slab, b = 1000 m

w d = γ c bh
Dead load:
w d=23.5 (1)(0.16)

wd=3.76 kN−m

Effective depth: d = 160 – 20 – 1/2(12)


d = 134 mm

12
¿
Steel ¿
area, 1000 1000 π
As = x As As= x ¿
s 140 24

As =807.8 mm
2

As 807.8
ρ= ρ=
bd 1000( 134)

ρ=0.006028
0.85 f ' c β 1600 ( 20.7) ( ) (600)
= = 0.85 0.85
b b
ρ f y ( 600+f y ) ρ 275 (600+275)

ρb=0.037> ρ( steel yields )

ρf 0.006028( 275)
ω= ω=
yf ' c 20.7

Rn=f 'c ω (1−0.59 ω )Rn =20.7 (0.0801)[1−0.59 (0.0801)]

Rn=1.58 MPa

Mu=φ Rn bd Mu =20.7 (0.0801)[1−0.59 (0.0801)]


2

Mu=25.5334 kN −m

2 2
wu L wu (3)
M = 25.5334=
u
8 8

wu
= 22.696 kN/m

wu=1.4 wDL+ 1.7 w¿ 22.696=1.4 (3.76)+1.7 wL

wL=10.25 kN / m
w¿=Uniform pressure xb

10.25 = Uniform pressure x 1


Uniform live load pressure = 10.25

kPa Solved Problems Using 2010 NSCP

PROBLEM 2.29

A reinforced concrete beam has width of 310 mm and an effective depth of 490 mm.
, . Determine the following:
a) The balanced steel area
b) The maximum steel area for singly reinforced condition
c) The maximum design strength if the beam is singly reinforced
d) The required steel area if the beam is subjected to dead load moment of 120 kN-
m and live load moment of 170 kN-m.

SOLUTION
f ' c>28 MPa;
Since
0.05 0.05
(f −28 ) β =0.85−
'
β =0.85− (30−28)
1 c 1
7 7

β1=0.836

a) Balanced steel area:


'
0.85 f β 600
c 1 0.85 (30) (0.836 ) (600 )
ρ= ρ=
b
f y (600+ fy ) b
415 (600+415 )

ρb=0.03036
A sb= ρb b d A sb=0.03036 (310)( 490)

Asb=4,611 mm
2

b) Maximum steel area when beam is singly reinforced:


From Eq. 2-24: ρmax =
'
3 0.85 f c β 1
7 fy

3 0.85 (30 ) ( 0.836 )


ρmax =
7 415 (600+415 )

ρmax =0.0221

A s max =ρmax b d A s max =0.0221(310 )( 490)

Asmax =3,343 mm
2

φ M n max : ε =0.004, f s =800 MPa


c) Maximum design strength,

M n max 51 β 1 f 'c bd 2 1 3 β1
From Eq. 2-25 : = ( − )
140 14

2 3
490 ¿ (1− x 0.836)
14
51
M = (0.836 ) (30 ) (310) ¿
n max
140
Mn max=558.05 kN −m

φ=0.65+0.25 800−f y
From Eq. 2- 1000−f y
26:

800−415
φ=0.65+0.25
1000−415

φ=0.8145

φMn max=0.8145(558.05)

φ Mn max=454.55 kN −m

M u=1.2 M D +1.6 M L M u=1.2(120)+1.6(170)


d)
Mu=451.45 kN −m

Thus, the beam is singly reinforced.

Determine if the beam is tension-controlled:


459 2 3
φ M tn= β f ' bd (1− β )
From Eq. 2-22: 1600 1 c 16 1

φM tn=451.45 kN −m

M u is less than M tn ,
Since the required the section is tension controlled.
φ=0.90
Mu=φMn a
Mn=φ x 0.85 f 'c ab(d − )
2

6 a
416 x 10 =0.90 x 0.85(30)(a)(310)(490− )
2

a=139.06 mm

Check if it is really tension-controlled:


a 139.06
c= = =166.4 mm
β1 0.836

d−c 490−166.4
f =600 =600 =1,167 MPa>1,000 MPa(OK )
s
c 166.4

PROBLEM 2.30

Given the following data for a rectangular beam: width , effective depth

, , . Dead load moment

Live load moment

SOLUTION
β1=0.85

M U =1.2 M D +1.6 M L M u=1.2(180)+1.6(167)


Mu=483.2 kN −m

φM n max
Solve for to determine if compression steel area is required.

M n max= 140 β 1 f 'c bd 1− 14 β 1


)
520
¿
¿
51
Mn max = (0.85 ) (27 ) (320) ¿
140

Mn max=591.64 kN−m

800−f y
φ=0.65+0.25
1000−f y =0.8237

φM n max=487.31 kN−m> M u (singly reinforced )

φM tn
Solve for to determine if the section is tension-controlled.

)
φM n= 1600 β 1 f ' c bd 1− 16 β 1 =478.9 kN −m

M u > φM tn φ<0.90
Since , the section is within “transition region’, i.e 0.65 <

Mu=φMn=φx 0.85 f ' c ab( d−a /2)


f s −f y 520−c
φ=0.65+0.25 600 −345
1000−f =0.65+0.25 c
y 1000−345

119.084
φ= +0.2893
c

φ=0.85 c

( )
6 119.084
483.2 x 10 = c + 0.2893 x 0.85( 27)(0.85 c)(320)(520−1/ 2 x 0.85 c)

c=208.8 mm

a=β1 c=177.45 mm

T =C As f y=0.85 f ' c ab

A s (34.5)=0.850(27 ) (177.45) (320)

2
As =3,777 mm
PROBLEM 2.31
Given the following properties of a rectangular concrete beam: b = 280 mm, d = 480
mm, , . The beam is reinforced for tension only.
Determine the design strength under the following conditions.
When the beam is reinforced with three 25 mm diameter bars.
When the beam is reinforced with four 25 mm diameter bars.
When the beam is reinforced with seven 25 mm diameter bars.

SOLUTION

β1=0.85 since f ' c is less than 28 MPa


'
0.85 f β 600 0.85 (21) (0.85 ) (600)
c 1
ρ= ρ=
b
f y (600+ fy ) b
415 (600+415 )

ρb=0.0216

π 2 2
A= (25) =490.87 mm
b
4

a) A s =3 x Ab =1473 mm2

As 1473
ρ= ρ=
bd 280 (480)

ρ=0.01096< ρb ( steel yields)


'
C=T 0.85 f c ab=A fs
y

0.85 (21) (a ) (280 )=1473( 415)

a=122.28 mm

a
c= =143.86 mm
β1

d−c 480−143.86
f =600 =600 =1,402 MPa>1,000 MPa
s
c 143.86

φ=0.90
The section is tension-controlled,

M n=C c (d−a/ 2) M n =0.85 f ' c a b(d−a /2)

M n=0.85 (21) (122.28 ) (280)(480−122.28 /2)

Mn=255.87 kN −m

φMn=0.90( 255.87)

φMn=230.28 kN −m

2
b) A s =4 x Ab=1963 mm
As 1963
ρ= ρ=
bd 280 (480)

steel yields
ρ=0.014961< ρb ¿ )

C=T 0.85 f ' c ab= As f y

0.85 (21) (a ) (280 )=1963(415)

a=163.04 mm

a
c= =191.81 mm
β1

d−c 480−191.81
f =600 =600 =901.5 MPa< 1,000 MPa
s
c 191.81

φ<0.90
The section within” transition region”, i. e 0.65 <
f s−f y
φ=0.65+0.25 901.5−415
1000−f y φ=0.65+0.25
1000−415

φ=0.858

M n=C c (d−a/ 2) M n =0.85 f ' c a b(d−a /2)

M n=0.85 (21) (163.04 ) (280 )(480−163.04/ 2)


Mn=324.504 kN −m

φMn=0.858( 324.504)

φMn=278.396 kN −m

A s =7 x Ab =3436 mm
2
c)

As 3436
ρ=
bd = 280 (480)
ρ

ρ=0.02557> ρb ( steldoes not yield )

φ=0.65
The section is compression-controlled,

T =C As f s=0.85 f 'c ab

480− c
3436 x 600 =0.85 (21 ) (0.85 c )(280)
c

c=297.56 mm

a=β1 c=252.92 mm

M n =C c ¿ d−a/ 2 ¿ M n =0.85 f c ab(d−a/ 2)


'

M n=0.85 (21) (252.92) (280)( 480−252.92/ 2)

φMn=0.65( 446.91)
φMn=290.49 kN −m

PROBLEM 2.32

A hallow beam is shown in Figure 2.16. Assume and .

Calculate the required tension steel area when


What is the balanced moment capacity of the beam?
What is the maximum steel area under singly reinforced condition?
What is the maximum design moment strength under singly reinforced condition?
Calculate the required tension steel area when .

500 mm
125250125
800 mm
500
150
15
0

75 mm

Figure 2.16 - Hallow beam


SOLUTION

This problem is the same as Problem 2.23.

d=800−75=725 mm

To guide us whether “a” will exceed 150 mm or not, let us solve the design moment
when a =150 mm.
a
c= =176.47 mm
β1

d−c
f =600 =1,865 MPa>1000 MPa Tension controls ,φ=0.90
s
c

φM n=φ C c (d −a/ 2)φ Mn=0.90 x 0.85 (28 ) (150) ( 500 ¿(725−150/ 2)

φMn=1044.225 kN
−m

Mu=800 kN −m
a)
Mu=800 kN −m <1044.225 kN−m, a<150
Since the required

mm. M u=φ M n M u =φ C c ( d−a/ 2)

Mu=φ 0.85 f ' c ab (d−a/ 2)


800 x 10 =0.90 x 0.85 (28) a(500)(725−0.5 a)
6

a=111.6 mm<150 mm

Stress in steel
d−c a
f =600 where c = =131.3 mm
s
c β1
725−131.2
f =600 =2,712 MPa>f
s 131.3 y steel yields

T =Cc As f y=0.85 f ' c ab

A s (345)=0.85(28 ) (111.6)(500)

As =3,850 mm
2

φ=0.65
b) Balanced condition:

600 d 600(725)
C= C= =460.32mm
b b
600+f y 600+345

a=β1 c b a=0.85 (460.32)=391.3 mm


500 mm
125 250
125

1 15
0
2 2
a
72
5

Figure 2.17
N
z=a−150=241.27 mm

A 1=500 ( 150 )=75,000 mm y 1=725−1/2(150)=650 mm


2

¿
A 1=125 ( 241.27 ) =30,159 mm
2
y2=725 −150−1/ 2 241.27) = 454.37
¿

M bn=C1 y1+ 2C2 y2

M bn=0.85 f' c ( A1 y1+ 2 A2 y2)

M bn=0.85 (28)[ 75,000 x 650+2 x 30,159 x 454.37 ]

M bn=1812.52 kN−m

φM bn=0.65 x 1812.52

φ M bn=1178.14 kN−m

A s max Refer ¿ Figure 2.17


c) Maximum steel area,

Cmax 3
= d=310.71 mma=β c =264.11 mm
1 max
7
z=a−150=114.11 mm

A 1=500 ( 150 )=75,000 mm y 1=725−1/2( 150)=650 mm


2

A 2=125 ( 114.11 )=14,263 2 y2=725−150−1/ 2


(114.11)=517.95
mm

T =C1 +C 2 As max f y=0.85 'f c ( A1 +2 A2)

A s max (345 )=0.85 (28)[ 75,000 x 650+2 x 14,263]

As max =7,142 mm
2

M n max
d) Maximum moment, :

Mn max=C 1 y1 +2 C2 y2

Mn max=0.85 f 'c +( A1 y1 +2 A2 y2 )

M n max=0.85(28 )[75,000 x 650+ 2 x 14,263 x 517.95]

Mn max=1511.9 kN−m
800−f y
φ=0.65+0.25 =0.824
y
1000−f
φ Mn max=0.824 x 1511.9

φM n max=1245.3 kN−m

e) Mu=1200 kN −m< φ M n max (singly reinforced )

Refer to Figure 2.17

Mu=φ 0.85 f c ( A1 y1 +2 A2 y2 )
'

f s+ f y d−c
f =600
φ=0.65+0.25 y s
1000−f c

725−c
600 c −345 166.03
φ=0.65+0.25 = +0.28 93
1000−345 c

z=a−150=0.85 c−150

A2=125 z=106.25 c−18,750

y2=725−150−1/ 2 z
=575-1/2(0.85c-150)

y2=650−0.425 c
Mu=φ 0.85 f c ( A1 y1 +2 A2 y2 )
'

( )
166.03
+0.2893 0.85 (28) [75,000 ](650)
6
1200 x 10 = c

+2(106.25 c−18,750)(650−0.425 c)¿

c=398.7 mm

166.03
φ= +0.2893=0.706
398.7

A 2=106.25 ( 398.7 ) −18,750=23,615 mm


2

T =C As f y=0.85 f ' c( A1+2 A2 )

A s (345)=0.85(28 )(75,000+2 x 23,615)

As =8,432 mm
2

PROBLEM 2.33

Design a singly reinforced rectangular beam to carry dead load moment of 110 kN-m
(including self weight) and live load moment of180 kN-m. Use steel ratio

and take Assume and

SOLUTION
M u=1.2 M D +1.6 M L M u=1.2(110 )+ 1.6(180)

Mu=420 kN−m

0.85 f ' c β1600


ρ= =0.03765 Note : β =0.85 since f ' < 28 MPa
b
f y ( 600+f y ) 1 c

ρ=0.65 ρb=0.02447

ρfy
ω= c =0.322
f'

Rn=f c ω (1−0.59 ω)¿=5.473 MPa


'

600 d
C= C =0.685 d
b b
600+f y

ρ
Note: For singly reinforced rectangular beam, is directly proportional to c.

c=0.65 c b
Thus,
c=0.445 d
d−c
f 600
= f s =600 d −0.445 d
0.445 d
s
c

f s=747.7 MPa<1000 MPa transition


f s−f y 747.7−276
φ=0.65+0.25
φ=0.65+0.25 y 1000−276
1000−f

φ=0.813
M u=φ Rn b d 420 x 10 =0.813 (5.473 ) ( b ) (1.9 b)
2 6 2

b=297 mm

d=1.9 b=564 mm

A s =ρbd As =0.02447 (297) (564)

As =4,1000 mm
2

PROBLEM 2.34

Repeat Problem 2.33 using a steel ratio

SOLUTION
Mu=420 kN−m

ρb=0.03765

ρ=0.5 ρb=0.01883

ρf y
ω= (1−0.59 ω)=4.438 MPa
'
f c
600 d
C= C =0.685 d
b b
600+f y

c=0.5 cb =0.34247 d
d−c
f 600
= d−0.34247 d
f s =600 d− 0.324247 d
s
c
f s=1152 MPa>1000 MPa, φ=0.90

M u=φ Rn b d 420 x 10 =0.90 ( 5.473 ) ( b ) (1.9 b)


2 6 2

b=308 mm

d=1.9 b=585 mm

A s =ρbd A s =0.01883 (308)(585)

As =3,390 mm 2

SUPPLEMENTARY PROBLEMS
PROBLEM 2.35

A rectangular beam has .


Determine (a) the maximum design moment if the beam is singly reinforced and (b) the
required steel area if the beam is required to carry a dead load moment of 50 kN-m and
a live load moment of 30 kN-m. Use the 2001 NSCP.

Answer : a ¿ φMn max=148.3 kNm

b ¿ As =1075 mm
2

PROBLEM 2.36

Repeat Problem 2.35 using the 2010 NSCP.

Answer : a ¿ φMn max=130.8 kN−m

b ¿ As =1056 mm
2

PROBLEM 2.37

Design a rectangular beam reinforced for tension only carry dead load moment of 85
kN-m (including its estimated weight) and a live load of 102 kN-m. Use and

use d= 1.75b. Assume and . Use the 2001 NSCP

Answer : b=250 mm, d=436 mm, As=3,273 mm 2

PROBLEM 2.38

Repeat Problem 2.37 using the 2010 NSCP.


Answer : b=246 mm, d=430 mm, As=3182 mm 2

PROBLEM 2.39

A reinforced concrete beam has the following properties: Use 2001 NSCP)
beam with,
effective depth,

concrete

strength,

reinforcing steel,

reinforcing steel modulus,

service dead load moment


a) If the beam is to be designed for a balanced condition, find the required area of
steel area reinforcement, design balanced moment, and the corresponding
service live load moment.
b) Find the maximum steel area, the maximum design moment, and the
corresponding service live load moment if the beam is to be designed as singly
Answer : a ¿ Asb=5,549mm , φMn=952.44, ML=272 kN −m
2

b ¿ As max =4,162mm , φ Mn max =775.46, ML=168 kN −m


2

PROBLEM 2.40

Repeat Problem 2.39 using the 2010 NSCP.


2
Answer : a ¿ 5,549 mm ,φ M
n =687.87 kN−m, M =167.42 kN−m

b¿A
=677,7 =161 kN −m
2
s =3,963 mm , φ n L
max
M max
M
PROBLEM 2.41

Calculate the ultimate moment capacity of a rectangular beam with ,

, . Assume . . Use 2001


NSCP

Answer : φMn=366.2 kN −m

PROBLEM 2.42

Repeat Problem 2.41 using the 2010 NSCP.

Answer : φ Mn=366.2 kN −m

PROBLEM 2.43

Calculate the ultimate moment capacity of a rectangular beam with ,

, . Assume , . Use 2010


NSCP.

Answer : φ Mn=582.9 kN −m

PROBLEM 2.44

Repeat Problem 2.43 using the 2010 NSCP.

Answer : φMn=514.3 kN −m

PROBLEM 2.45
Calculate the ultimate moment capacity of a rectangular beam with ,

, . Assume , . Use 2010


NSCP

Answer : φ Mn=729.6 kN −m

PROBLEM 2.46

Repeat Problem 2.45 using the 2010 NSCP.

Asnwer : φ Mn=522.5 kN −m

CHAPTER 3

Analysis and Design of T-Beams


and Doubly Reinforced Beams
T-Beams
Reinforced concrete floors usually consist of slab and beams, which are placed or
poured monolithically. In this effect, the beam will have extra width on top (which is
usually under compression) called flangers, and the resulting section is called a T-
beam. The beam may also be L-shaped if it is located at the end of slab.

ANALYSIS AND DESIGN OF T-BEAMS


WITH FLANGE IN COMPRESSION

Because of the huge amount of compression concrete when the flange of a T-beams is
compression, the section is usually tension-controlled (extreme tension yields).

The compression block of T-beam may fall within the flange only or partly in the web. If
it falls within the flange as shown in Figure 3.1 (a), the rectangular beam formulas in
Chapter 2 applies since the concrete below neutral axis is assumed to be cracked and
its shape has no effect on the flexure calculations. If however it cover part of the web as
shown in Figure 3.1 (b), the compression concrete no longer consist of a single
rectangle and thus the rectangular formulas do not apply.

BALANCED AND MAXIM(aUM) STEEL AREA(bA)ND MOMENT


The balance d v alu e o f “c” fo r an y be am sh a p e ,
F i g u r e 3 . 1 : L o ca t i o n
as d isc u ss ed i n C ha p te r 2 is given by:
o f n e u tr a l a x i s
600 d
Cb=
600+f y
ab =β 1 C b
and

If ”a” is less than the slab thickness, the formulas for rectangular beam may be used, or

0.85 f ' c β1600


ρb= f ( 600+f )
y y

Asb= ρb br d
M
' =0.85 f a (d −ab )
bn c b
2

As max =0.75 Asb

amax =0.75 ab

Mn max c
amax
=0.85 f amax b(d− 2 )
'

However, if “a” is greater than the slab thickness, the following formula will be used.

t a
Z d

T =C1 +C 2 Asb f y=C 1+ C2


Asb f y=0.85 f c (b f t +b w z )
'

0.85 f 'c [ f' c t +( a−t) bw]


Eq. 3-1 Asb=
fy

As max =0.75 Asb


Eq. 3-2

DESIGN OF T-BEAMS WITH NEGATIVE MOMENTS

N.
A

When T-beams are resisting negative moments so that far their flangers are in tension
and the bottom of their stems in compression, the formulas for rectangular beams can
be applied. The following code requirements shall be applied for this case:
410.7.6: Where flangers of T-beam construction are in tension, part of the flexural
tension reinforcement shall be distributed over an effective flange width as defined in
Sec. 408.11, or width equal to 1/10 the span, whichever is smaller. If the effective flange
width exceeds 1/10 the span, some longitudinal reinforcement shall be provided in the
outer portions of the flange.

The intention of this section is to minimize the possibilities of flexural cracks that will
occur at the top face of the flange due to negative moments.
MINIMUM STEEL RATIO

For statically determinate T-section with flange in tension, the minimum steel area is
equal to or greater than the smaller value of Eq. 3-3 and Eq. 3-4:

As =
√f 'c b d
Eq. 3-3 min 2 fy w

As =
√ f 'c b d
Eq. 3-4 min 4 fy f

CODE REQUIREMENTS ON T-BEAMS (SECTION 408.11)


NOTE: THESE REQUIREMENTS ARE THE SAME WITH 2010 NSCP

1. In T-beam construction, the flange and web shall be built integrally or otherwise
effectively bonded together.

2. The width of slab effective as T-beam shall not exceed 1/4 of the span of the
beam, and the effective overhanging flange on each side of th web shall not
exceed:
a) 8 times the slab thickness, and
b) 1/2 the clear distance to the next web.

3. For beams with slab on one side only, the effective overhanging flange shall not
exceed:
a) 1/12 the span length of the beam,
b) 6 times the slab thickness, and
c) 1/2 the clear distance to the next web.

Interior Beam end Beam


Figure 3.2: Effective flange width
For Interior Beam

bf
is the smallest
of: 1. L/4
16 t + bw
2. S1 S2
+ +b
3. 2 2
w

For End Beam

b'f
is the smallest of
b'w
: 1. L/12 +

6 t + b 'w
2.
S3 / 2+b' w
3.

S1=S2=S ¿
For symmetrical interior beam (
bf
is the smallest of:
1. L/4
16 t + bw
2.
3. center-to-center spacing of beams
4. Isolated beams in which T-shape are used to provide a flange for additional
compression area shall have a flange thickness not less than 1/2 the width of the
web and an effective flange width not more than four times the width of the web.

t ≥b w / 2 t
bf ≤ 4 bw

5. Where primary flexural reinforcement in a slab that is considered as a T-beam


flange is parallel to the beam, reinforcement perpendicular to the beam shall be
provided in the top of the slab in accordance with the following:
a) Transverse reinforcement shall be designed to carry the factored load on
the overhanging slab with assumed to act as a cantilever. For isolated
beam, the full width of the overhanging flange shall be considered. For
other T-beams, only the effective overhanging slab needs to be
considered.
b) Transverse reinforcement shall be spaced not further apart than five times
the slab thickness, nor 450 mm.

Transverse reinforcement
To be provided

Primary slab reinforcement


AS
STEPS IN FINDING THE TENSION STEEL AREA OF SINGLY
MU
REINFORCED T-BEAMS WITH GIVEN AND OTHER BEAM
PROPERTIES:

φ Mn
I. Solve for max to determine of compression steel is necessary.

Follow the procedure in Page 105.

Mu≤φ Mn max
If , the beam is singly reinforced, proceed to Step II.
M u > φ M n max
If , the beam is doubly reinforced

Note : SKIP stepif ∈ your judgment Muis small∧¿

compression steel is not needed .


φM n 1
II. Solve for when a = t

C=0.85 f ' c b f
Compressive force in concrete,

t
M n 1=φ C (d−t / 2)
¿
¿
t
( )= ¿
'
φ M n 1=φ 0.85 f c bf d− ¿
2

φM n 1> M u , then a< t , proceed ¿ Step III


if
φM n 1> M u , then a> t , proceed ¿ Step IV
if
a<
III.
t
t a
d d
-a/2

Solve for a:

a
Mu =φ∁(d− )
=φ n
2
M

M u =φ 0.85 f ' c ab (d−a/


2)
¿
¿
¿
¿
a=¿ ¿

T =C As f y=0.85 f ' c ab

¿
¿
As =¿ As min

As min
is the smaller value of:
√ f 'c √ f 'c
A = b d A = bd
s w smin f
min 2fy 4fy

IV.

t
a
Z d

M u =φ M n M u =φ M n 1+ M n 2 Note : φ Mn is∈Step
¿
¿
¿
¿
Mn 2=¿ ¿
Mn 2=C2 y2=0.85 f ' c bw z y2
¿
¿
¿
¿
¿
z=¿ ¿
T =C1 +C 2 A s f y =C 1 +C2
As f y=0.85 f 'c (bf t + bw z)
¿
¿
¿
As =¿ ¿

As min
is the smallest value of:

√ f ' c=
A √ f 'c
b d A = bd
s w s f
min 2fy min 4fy

φM n As
STEPS IN FINDING OF SINGLY REINFORCED T-BEAMS WITH GIVEN
AND OTHER BEAM PROPERTIES:

Asb
I. Solve for balanced steel area to determine if tension steel
yiel. Follow the procedure in Page 105.

A s ≤ Asb
If ,tension steel yields. Proceed to step II
A s > A sb
If , tension does not yield

Note : This stepmay skipped if ∈ your judgement A s is

small∧¿ instead assume that f s =f y


.

f s=f Ac
II. Tension steel yields, y
. Compute the area of compression concrete,

C=T 0.85 f ' c Ac= As f y

¿
¿
Ac =¿ ¿
Ac A f =bf t
Compare with the area of compression flange,

A s < A f , a< t ,
If proceed to Step
III A c > Af , a>t ,
If
proceed ¿ Step IV

a<t
III.

Solve for a: a
t d d -a/2
Ac=bf x a
¿
a=¿ ¿

φM n=φT (d−a/ 2)
a
φMn (d − )
=φ As y
2
f

a >t
IV. :

t a
Z d
T
z
Solve for :
Ac= A1+bw z
¿
¿
¿
z=¿ ¿

φMn=φMn 1 +φMn 2

φMn=φ (C1 y1+ C2 y2)

φM n=φ 0.85 f ' c [ A 1 y1 + A2 y2 ]

ILLUSTRATIVE PROBLEMS

SOLVED PROBLEMS IN T-BEAMS USING 2001 NSCP

PROBLEM 3.1
Determine the effective flange with for symmetrical T-beam with a span of 6 m. The
beam width of web is 250 mm, the slab thickness is 120 mm, and the clear distance to
adjacent beams is 3m.
SOLUTION
For symmetrical T-beam, the effective flange width is the smallest of:
1. 1/4 span = 6000/4 = 1500 mm
16 t + bw =16 (120)+250=2170 mm
2.
bw=3000+250=3250 mm
3. clear spacing of beams +

bf =1500 mm
Therefore

PROBLEM 3.2
Given the following elements of a T-beam:
Flange width,

Flange thickness,

Width of web,

Effective

depth,
If the beam is reinforced for tension only, determine the ultimate moment capacity when
the depth of compression concrete flange equals the flange thickness or .
SOLUTION
a
Mn=0.8 f ' c a( d− )
bf 2

When a=t

Eq. 3-5 Mn=0.85 f c' bf t (d −t /2)

M fn 180
=0.85 (30) (120 ) (130)( 470− )
2

M fn=1611 kN −m

φ Mn=0.90 x 1611=1450 kn−m

PROBLEM 3.3
Given the following elements of a T-beam:
Flange width,

Flange thickness,

Width of web,
Effective depth,
If the beam is reinforced for tension only, determine the following:
The balanced steel area
The nominal and ultimate balanced moment capacity
The maximum steel area
The nominal and ultimate maximum moment capacity

SOLUTION
β1=0.85 since f ' c is less than 30 MPa

a) Balanced condition

600 d 600 ( 460 )


C= C=
b b
600+f y 600+
414
Cb =272.2 mm

a=β1 c a=0.85(272.2)

a=231.4 mm>t

= 900mm
t=100

C a
z
d = 460
mm

=250 mm
Figure 3.3
z=a−t =121.4 mm

A 1=b f x t =900 ( 110 )=99,000 mm


2

A2=bw x z=310 (121.4)=37,622mm


2

2
Acb =A1 + A2=136,622 mm

T =C1 +C 2 Asb f y=0.85 f c ( A1 + A2 )


'

A sb (414 )=0.85 (20.7)99,000+37,622 ¿

Asb=5,806 mm →balanced steel area


2

y1=d−t / 2=405 mm

y2=d −t −z /2=289.3 mm

M bn=C2 y1+ C2 y 2 M bn=0.85 f c( A 1 y1 + A 2 y2)


'

M bn=0.85 (20.7)[ 99,000 (405)+37,622 (289.3)]

M bn=597 kN −m→nominal balance d moment

M bn=0.90(897)

M bn=807.3 kN −m→ ultimate balanced moment

b) Maximum steel area and moment. Refer to Figure 3.3.


As max =0.75 Asb Asmax=0.75(5806)

As max =4,355 mm →maximum steel area


2

A c max =0.75 A cb A c max =0.75(136,622)

Ac max =102,466 mm > A1 ,thus a>t


2

A c max = A 1 + A 2
102,466=99,000 + 310(z)
z=11.2 mm

A2=102,466−99,000=3,466 mm
2

z
y =d −t − =344.41 mm
2
2

Mn mnx=C1 y1 +C2 y2 Mn max=0.85 f 'c( A1 y1 + A2 y2)

M n max=0.85(20.7 )[ 99,000( 405)+3,466 (289.3)]

Mn max=726.5 kN −m →nominal max moment

M n max=0.90(726.5 )

Mn max=653.8 kN−m→ultimate maximum moment

PROBLEM 3.4
A T-beam has the following properties: , ,

Concrete compressive strength and steel


area for the following load conditions:
a)
b)

SOLUTION
β1=0.85
Solve for ↑φ Mn when a=t
t
( )
φM 0.85 f ' b t d 545.375 kN mm
fn = c f −2 = −

Solve for φMn max :

Balanced condition:
600 d
c= =278.11 mm
b
600+ f y

a< β1 cb=236.39 mm>t

= 820mm
t=100

C a
d = 470

z
mm

=250 mm

FIGURE 3.4
z=a−t =136.39 mm
2
A1=bf t =82,000 mm
A2=bw z =34,098 mm
2

Acb =A1 + A2=116,098 mm


2

Maximum condition:
A c max =0.75 A c b=87,073 mm >
2

2
A1 A 2 = Acmax −82,000=5,073 mm
As
z= =20.29 mm
bw
z
y =d −t − =359.85 mm
2
2

φ M n max= Mn + M n 2=M fn +0,85 f ' c A2 y 2

φM n max=574.28 kN−m

M D =150 kN −m, AML=120 kN−m


a)
Mu=1.4 MD +1.7 ML=414 kN−m< φMn max , singly reinforced

M u is less than M fn , a is less than t.


Since

t =820
=100
mm C
d = 470

d -a/2
mm

T
Mu=0.85 f ' c abf (d−a /2)

414 x 10 =0.90 ( 0.85 ) ( 20.7 ) a ( 820 ) ( 470−a/ 2)


6

a=73.6 mm

T =C A s f y=0.85 f ' c ab f

As =2,565 mm
2

As
Minimum is the smaller of:

√f'c
√f'c
2
d=646 mm d=1059 mm
2

b
b2 f w
4fy
f
y

A s =2,565 mm
2
Thus,

M D =175 kN −m, ML=190 kN −m


b)
Mu=1.4 MD +1.7 ML=568 kN−m< φMn max , singly reinforced

Mu M fn , a
Since is more than is more than t.
= 820mm
t=100

C a
z
d = 470
mm

T
=250 mm
Mu=φM fn +φMn 2
6 100 z
568 x 10 =545.375+ 0.90 (0.85) (20.7 ) (250) z (470− )
2
z=15.78 mm

A2=bw z =3,946 mm 2

T =C1 +C 2 As f y=0.85 f c( A1+ A2)


'

A s (414 )=0.85 (20.7 )( 82,000+3946)

As =3,653 mm2

PROBLEM 3.5
Design a T-beam for a floor system for which and . The
beams are 4.5 m long and spaced at 3 mo.c. The slab thickness is 100 mm.

SOLUTION
β1=0.85
M u=1.4 M D +1.7 M L M u=1.4 (450)+1.7(350)

Mu=1225 kN −m

Solve for bf:


bf is the smallest of :
1. L/4 = 1.125 m
16t + bw =16 (100)+300=1,900 mm
2.
3. center-to center spacing of beams = 3

m bf =1,125 mm
Thus,

φ M n whena=t =100 mm, φ=0.90


Solve for t
φ M =φ 0.85 f ' t b (d− )
fn c f
2
φMn=1161.844 kN −m

φM n max
Solve for to determine if compression steel is needed.

600 d
c= =325.123 mm
b
600+ f y

a=β1 cb=276.355 mm> t

= 1125mm
t=100
d =550mm

M u=φMC fn +aφ
z
c2 y2
450

=300 mm
6 6 z
1225 x 10 =1161.844 x 10 + 0.90 x 0.85(27)(300 z)( 450− )
2
z=23.25 mm

A2=bw z =6975.02mm 2

T =c1+ c2 A s f y=0.85 f c ( A1 + A2 )
'

A s (415 )=0.85 (27)(112,500+ 6,975.02)

As =6,607 mm
2

As
Minimum is the smaller value of:
√f ' c
√f'c
2
d=1033 mm d=1937 mm
2

b
b2 f w f
y 4fy

A s =6,607 mm
2
Thus,

PROBLEM 3.6
Determine the ultimate moment capacity of reinforced concrete T-beam with the
following properties: Flange width b = 1500 mm, web width , effective

depth d = 600 mm, slab thickness t = 100 mm. Assume and

. The beam is reinforced with six 28 mm bars.

SOLUTION
= 1500mm
As : t=100
Solve for balanced
d =600mm

C a
z

=250 mm
600 d
c= =380.95 mm
b
600+ f y

a=β1 cb=323.81>t

z=a−t =22381 mm

A1=bf t =150,000

A2=bw z =55,952mm
2

T =C A s f y=0.85 f c ( A1 + A2)
'

A sb (345)=0.85 (20.7 )(150,00+ 55,952)

Asb=10,503

π 2 2
A s =6 x (28) =3,695 mm > Asb steel yields
Steel area provided, 4

f s=f y
Therefore,
C=T f ' c Ac= A s f y
0.85
Ac=3,695(345)
0.85(20.7)
Ac=72,441< A1 therefore a is less than t

t =1500
=100
mm C
d = 600

d -a/2
mm

T
A c = ab f
72,441 = a (1500)
2
a=48.29 mm

a
M =0.85 f ' ab (d− )
n c
f 2
48.29
M =0.85 (20.7) (48.29) (1500)(600− )
n
2
Mn=733.99 kN −m

φMn=0.90( 733.99)

φMn=660.6 kN
−m

PROBLEM 3.7
Given the following properties of T-beam:
Flange width,
Flange thickness, t=1200

Width of web,

Effective depth, d = 580 mm


Service deal load,
Determine the safe service live load if the beam is reinforced for tension only with
twelve (12) 28-mm-diameter bars.
SOLUTION
= 900mm
β1=0.85 ; φ=.90 t=120

d =580mm
28
¿ C a
¿ π z
A =12 x ¿
s
4

A1=bf t =108,000 mm 2
A
Solve for balance s :
600 d
c= =368.25 =400 mm
mm
b
600+ f y

c=β1 cb=313.02 mm>t

z=a=t =193.02 mm

A2=bw z =77,206 mm
2

T =C Asb f y=0.85 f' c ( A1 + A2 )

A sb (345)=0.85 (21)(108,000+77,206)

Asb=9,582m m
2

Steel area provided is less than the balanced steel area. Steel yields.

C=T 0.85 f ' c A c = A s f y

0.85 (21) Ac=7,389(345)

Ac=142,813 mm > A1 a>t


2

= 900mm
t=120
d =580mm

C a
z
460

=400 mm
A c = A1+ A2 A2
142,813=108,000+
A2=34,813 mm 2

A 2 =b w z
34,813 = 400z
z=87.03 mm
t
y =d− =520 mm
1
2
z
y =d −t − =416.48 mm
2
2

Mn=C1 y1 +C2 y2

Mn=0.85 f c ( A1 y1+ A2 y2 )
'

Mn=0.85 (21)[ 108,000(520)+34,813 (416.48 )]

Mn=1,261.3 kN −m

φMn=0.90( 1,261.3)

φMn=1135.138 kN −m

φM n =M u Mu=1.4 M D +1.7 M L

1,135.138=1.4 ( 410)+1.7

M L ML=330.0 kN −m

PROBLEM 3.8
The section of a reinforced concrete T-beam is shown in Figure 3.5. The beam is
f y=415 MPa
reinforced with 10 32-mm-diameter tension bars with . Concrete strength
f ' c=32 MPa
. If the total service dead load moment on the beam is 330 kN-m,
determine the safe service live load moment.

=500m
m

d=530
t = 120mm

mm
.
10-32
SOLUTION mm
32
¿ =500mm
¿ π t =
A =10 x ¿ 120mm0mm a

d=530
s
=32
4
z

mm
2
A s =8,042 mm
5
Figure 3
A =b
2
t =60,000
mm
1 f

0.05
β =0.85− (32−30)
1
7
β1=0.836

As :
Solve for balance

600 d
Cb=
600+f y

Cb =313.3 mm

a=β1 Cb=261.83 mm>t


z=a−t =141.83 mm

A2=bw z =45,385.5 mm 2

T =C Asb f y=0.85 f' c ( A1 + A2 )

A sb (345)=0.85 (21)(60,00+ 45,385.5)

Asb=6,907 mm 2

A s > Asb
Since , tension steel does not yield
t = =500mm
120mm
d=530
a
z
mm

T
A 1 =60,000 mm
2 =320mm

A 2 =b w z =bw ( a−t
)

A 2 =b w ( β1 c−t )
d−c
f 600
s=
c

T =C1 +C 2 '
A s f s=0.85 f c( A1 + A2 )
530 −c
8,042 x 600 =0.85 (32)[ 60,000+ 320( 0.836 c−120)]
c
c=327.95 mm

A 2 =b w z =49,303 mm
2
a=β 1 c=261.83 mm

z
z=a−t =141.83 mm y2=d −t − =332.97
2 mm

y1=d−t / 2=470

M n =C 1 y1 +C2 y 2 Mn=0.85 f c ( A1 y 1 + A1 y2 )
'

Mn=0.85 (32)[ 60,000(470)+49,303 (332.97)]

Mn=1,213.56 kN −m

φ Mn=0.90(1,213.56)

φMn=1,092.2 kN −m

M u=φ M n Mu=1.4 M D +1.7 M L

1,092=1.4 (330)+1.7 M

L ML=370.7 kN −m
SOLVED PROBLEMS IN T-BEAMS USING 2010 NSCP

PROBLEM 3.9
Repeat Problem 3.3 using the 2010 NSCP.

SOLUTION

b f =900mm f ' c=20.7 MPa


Given:
t =110 mm f y=414 MPa

bw=3210 mm d=460 mm

β1=0.85 since f c is lesst h an 28 MPa


'

φ=0.65
a) Balanced condition,

600 d 600(460)
c b= c=b
600+ fy 600+414

cb=272.2 mm

a=β 1 c a=0.85(272.2)

a=231.4 mm>t

= 900mm
t=110

C a
z
d = 460
mm

T
=310 mm
Figure 3.6

z=a−t =121.4 mm

A 1=b f x t =900 ( 110 )=99,000 mm


2

2
A 2=b w x z=310 ( 121.4 )=37,622mm
2
A cb =A 1 + A2=136,622 mm

T =c1+ c2 Asb f y=0.85 f c ( A1 + A2


'

)
A sb (414 )=0.85 (20.7)(99,000+37,622)

Asb=5,806 mm 2→ balanced steel area

y1=d−t / 2=405 mm

y2=d −t −z /2=289.3 mm

M bn=c1 y1 +c 2 y2 M bn=0.85 f c ( A1 y1+ A 2 y2 )


'

M bn=0.85 (20.7)[ 99,000 (405)37,622 (289.3 )]

M bn=897 kN −m→nominal balanced moment

φM bn=0.65( 897)

φM bn=583 kN −m→ ultimate balanced moment

b) Maximum steel area and moment. Refer to Figure 3.6.


3 800−f y
c= d=197.14 mm; φ=0.65+0.25 =0.815
7 1000−f y

a=β 1 c a=0.85(197.14)

a=167.6 mm

z=a−t =57.571 mm

A 2=b w z =310 ( 57.6 )=17,847 mm


2

y2=d −t −z /2=321.21mm

T =c1+ c2 A s max f y=0.85 f c (A 1 + A 2 )


'

A s max ( 414 ) =0.85 f c (99,000+17,847)


'

Asmax =4966 mm →maximum steel area


2

Mn max=c1 y1+ c2 y2

Mnmax=0.85 f c (A1 y1 + A2 y2)


'

M n max=0.85(20.7 )[ 99,000( 415)+17,847 (321.2)]

Mn max=806.34 kN −m→nominal max moment

φMn max=0.815(806.34)

φMn max=656.9 kN−m→ultimate maximum moment

PROBLEM 3.10
Repeat Problem 3.2 using the 2010 NSCP.

SOLUTION
bf =1200 mm bw=290 mm
Given:
t =130 mm f ' c=30 MPa

d=470 mm f y=345 MPa


t
M =0.85 f ' t bf (d− )
fn c
2

M 130
fn
=0.85 (30) (1200 ) (130)( 470− )
2
M fn=1611 kN −m

φ
Solving for :

a=130 mm
0.05
(f −28 )=0.836
'
β =0.85−
1 c
7
a
c= =155.56 mm
β1
d−c
f =600 =1213 MPa>1000 MP tension-controls, φ=0.90
s
c
φM fn=090(1611)

φM fn=1450 kN −m

PROBLEM 3.11
Repeat Problem 3.4 using the 2010 NSCP.
Additional questions:
c) Find the required steel area if and .
d) Find the maximum design moment so that section is tension-controlled if it is
reinforced for tension only.
SOLUTION
b f =820 mm f ' c=20.7 MPa
Given:
bw=250 mm f y=414

MPa d=470 mm
t =100 mm

β1=0.85 since f ' c <28< MPa

φM n a= t :
Solve for when
M fn =0.85 f ' c b f t (d−t / 2)=605.97 kN−mm
a
c= =117.65 mm
β1

d−c
f =600 =1797 MPa>1000 MPa, φ=0.90
s
c

φ M fn=545.375 kN −m

φ M n max :
Solve for

Cmax 3
= d=201.43 mm
7

800−f y
φ=0.65+0.25 y =0.815
1000−f
a=β1 cmax =171.21mm> t
d = 470
mm

= 820mm
t=100

C a
A2=bw z =17,803.6 mm
2
z
y2=d −t −z /2=334.39 mm

Mn max=M
z=a−t fn+0.85 f ' c A2 y2
=71.21
T
mm
M n max=710.72 kN
=250 mm
−m φM n max=579

kN−m

M D =150 kN −m, ML=120 kN −m


a)
Mu=1.2 M D +1.6 ML=372 kN −m< φ Mn max , singly reinforced

Mu φM fn
Since is less than ,”a” is less than t.

t =820
=100
mm C
d = 470

d -a/2
mm

T
φ=0.90
Assume

a
M =φ 0.85 f ' ab ( d− )
u c f
2

6 a
372 x 10 =0.90 (0.85 ) (20.7) a (820)( 470− )
2
a=65.52mm

a
c= =77.08
mm β1
d−c
f =600 =3,058 MPa>1000 MPa, tension controls ,φ=0.90
s
c

T =C A s f y=0.85 f ' c ab f

A s (345)=0.85(20.7 ) (65.52)(820)

As =2,283 mm
2

As
Minimum is the smaller value of:

f'c 2 √f'c 2
d=646 mm b d =1059mm
b w f
2fy 4fy
Thus, As =2,283 mm
2

M D =175 kN −m, ML=190 kN −m


b)
Mu=1.2 M D +1.6 ML=514 kN−m<φ Mn max , singly reinforced

M u is less than φM fn ,a
Since is less than

t. φ=0.90
Assume

a
M =φ 0.85 f ' ab ( d− )
u c f
2

514 x 10 =0.90 ( 0.85 ) ( 20.7 ) a ( 820 ) (470−a /2)


6

a=93.53 mm

a
c= =110.03 mm
β1

t =820
=100
mm C
d = 470

d -a/2
mm

T
d−c
f =600 =1,963 MPa>1000 MPa ,tension controls, φ=0.90
s
c
T =C A s f y=0.85 f ' c ab f

A s (345)=0.85(20.7 ) (93.53 )(820)

As =3,259 mm
2

M D =195 kN −m, ML=210 kN −m


c)
Mu=1.2 M D +1.6 ML=570 kn−m <φ Mn max , singly reinforced

M u is more than φMu , a


Since is more than t.

= 820mm
t=100
d =470mm

C a
z

=250 mm

φ=0.90
Assume

Mu=φM fn +φMn 2
6 z
570 x 10 =545.375+ 0.90 (0.85) (20.7 ) (250) z (470−100− )
2
z=17.05 mm
a=t + z=117.05 mm; c=a / β1=137.7 mm

d−c
f =600 =1448 MPa>1000 MPa,tension controls ,φ=0.90
s
c

A2=bw z =3,908 mm
2

T =C1 +C 2 A s f y=0.85 f c ( A1 + A2)


'

As =3,666 mm2

3
c= d=176.25 mm, φ=0.90
d) b
a=β1 c=149.81 mm>t
t=100 = 820mm

C a
z
d = 470
mm

T
=250 mm

z=a−t =49.81 mm

A2=bw z =12,453mm
2

z
y =d −t − =3450.9 mm
2
2

M tn=M fn+0.85 f ' c A2 y2


M tn=681.59 kN−m

φM tn=613.4 kN −m

Mu φM n
Note: If is less than or equal to , the beam is tension-controlled.

PROBLEM 3.12
Repeat Problem 3.6 using the 2010 NSCP.

SOLUTION
b f =1500 mm f ' c=20.7 MPa
Given:
bw=250 mm f y=345 MPa

d=600 mm β1=0.85

As =6−28 mm=3,694 mm
2

As
Solve for balanced : = 1500mm
t=100
d =600mm

600 d
c= =380.95 mm C a
b
600+ f y
z
a=β1 cb=323.81>t

A1=bf t =150,000

A 2 =b w z =55,952 mm
2
=400 mm

T =C
'
Asb f y=0.85 f c ( A1 + A2 )
A sb (345)=0.85 (20.7 )( 150,000+ 55,952)

Asb=10,503

28
¿
¿
Steel area provided, π
As =6 x ¿
4

f s=f
therefore , y

C =T 0.85 f ' c A c = A s f y

0.85 (20.7) Ac=3,695(345)

Ac=72,441< A1 therefore a
is less than t

t =1500
=100
mm C
d = 600

d -a/2
mm

A c =ab f 72,441=a (1500)


2
a=48.29 mm

φ
Solve for :
a
c= =56.82
mm β 1

d−c
f s=600 =5,736 MPa>1000 MPa tension controls
c

φ=0.90
therefore

Mn=0.85 f ' c abf ( d−a /2)


48.29
M =0.85 (20.7) (48.29) (1500)(600− )
n
2
Mn=733.99 kN −m

φMn=0.90( 733.99)

φ Mn=660.6 kN −m

PROBLEM 3.13
Repeat Problem 3.7 using 2010 NSCP.

SOLUTION

Given the following properties of a T-beam:


bf =900mm f ' c=21 MPa
Flange width,
t =120 mm f y=345 MPa
Flange thickness,
bw=400 mm A s =7,389 mm
2
Width of web,
d=580 mm
Effective depth,

MD =410 kN −m
Service deal load,

β1=0.85 ; φ=0.90
2 2
28 ¿ =7,389 mm
t=120 = 900mm
π
A s=12 x ¿

d =580mm
4
C a
A1=bf t =108,000 mm
2
z

As
Solve for balance :

600 d
C = =368.25 mm =400 mm
b
600+f y

a=β1 cb=313.02 mm>t

z=a−t =193.02 mm
2
A2=bw z =77,206 mm

T =C Asb f y=0.85 f' c ( A1 + A2 )

A sb (345)=0.85 (21)(108,000+77,206)

Asb=9,582mm
2

Steel area provided is less than the balanced steel area. Steel yields.
C =T f ' c A c= A s f y
0.85
0.85 (21) Ac=7,389(345)

Ac=142,813 mm > A1
2
“a” >t

= 900mm
t=120

C a
z
d = 580

460
mm

T
=400 mm

A c = A1+ A2 142,813=108,000+ A 2

A2=34,813 mm
2

A 2 =b w z 34,813=400 z
z=87.03 mm

t
y =d− =520 mm
1
2
z
y =d −t − =416.48 mm
2
2

Mn=C1 y1 +C2 y2
'
Mn=0.85 f c ( A1 y1+ A2 y2 )
Mn=0.85 (21)[ 108,000(520)+34,813 (416.48 )]

Mn=1,261.6 kn−m

φ
Solve for :

a
a=t + z=203.03mm c= =243.57 mm
β1

d−c
f =600 =828.76 MPa<1000 MPa
s
c

f y< f s<1000 MPa,


Since Transition region
f s−f y
φ=0.65+0.25 =0.8346
y
1000−f

φMn=0.8346 (1,261.3)

φMn=1,052.703 kN −m

φM n =M u Mu=1.2 M D +.6 M L

1,052.703=1.2 (410)+1.7 M

L ML=350.44 kN −m

PROBLEM 3.14
Repeat Problem 3.8 using 2010 NSCP.

=500mm

d=530
m
t = 120m

mm
10-32 mm

=320mm

Figure 3.7
SOLUTION
2
32 ¿ =500mm
π t =120
A =10 x ¿
mm
s
4
a

d=530
2
A s =8,042 mm Z
2

mm
A1=bf t =60,000 mm

0.05
β =0.85− (32−28)
1
7
β1=0.821 =320mm

As
Solve for balanced :
600 d
c b=
600+ f y

a=313.3 mm
a=β1 cb=257.35 mm> t

z=a−t =137.35 mm

A2=bw z =43,953 mm 2

T =C Asb f y=0.85 f c ( A1 + A2 )
'

A sb (345)=0.85 (21)(60,000+ 43,953)

Asb=6,813 mm 2

A s > A sb ,tension steeldoes not yield


Since

t =500m
=120
mm m
a
d=530

Z
mm

=320m
m

φ=0.65 cpmression controls


A1=60,000 mm
2

A 2=b w z =bw (a−t )=bw ( β 1 c−t )

d−c
f =600
s
c

T =C1 +C 2 As f s =0.85 f c ( A1 + A2 )
'

530−c
8,042 x 600 =0.85 (32)[ 60,000+ 320(0.821 c−1200)]
c
c=329.27 mm

A 2 =b w z =48,151 mm
2
a=β 1 c=270.47 mm
z
z=a−t =150.47 mm y =d −t − =334.76 mm
2
2
y1=d−t / 2=470 mm

M n =C 1 y1 +c 2 y2 Mn=0.85 f c ( A1 y1+ A 2 y2 )
'

Mn=0.85 (32)[ 60,000( 470)+48,151 (334.76)]

Mn=1,205.48 kN −m

φMn=0.65( 1,205.48)

φ Mn=783.56 kn−m

M u =φM n Mu=1.2 M D +1.6 M L


78.56=1.2 (330)+1.6 ML

ML=242.23 kN −m

DOUBLY REINFORCED BEAM

Occasionally, beams are restricted in small sizes by space or aesthetic requirements to


such extent that the compression concrete should be reinforced with steel to carry
compression. Compression reinforcement is needed to increase the moment capacity of
a beam beyond that of a tensilely reinforced makes beams tough and ductile and
reduces long-time deflection of beams.

Compression steel also helps the beam withstand stress reversals that might occur
during earthquakes. Continuous compression bars are also helpful for positioning
stirrups and keeping them in place during concrete placement and vibration. Various
tests show that compression reinforcement also prevents the beam to collapse even if
the compression concrete crushes especially if it is enclosed by stirrups.

According to Section 407.12 of NSCP, compression steel in beams must be enclosed by


lateral ties, at least 10 mm in size for longitudinal bars 32 mm or smaller, and at least 12
mm in size for 36 mm and bundled bars. Deformed wire or welded wire fabric of
equivalent area is allowed. The spacing of these ties shall not exceed 16 longitudinal
bar diameters, 48 tie bar or wire diameters, or least dimension of the compression
member.

ANALYSIS OF DOUBLY REINFORCED BEAM


Mn
Doubly reinforced beam is analyzed by dividing the beam into two couples 1
and
Mn Mn
2
as shown in Figure 3.8. 1
is the couple due to compression concrete and
As 1 . Mu
the part of the tension steel 2
is the couple due to the compression steel
A 's As 2 .
and the other part of the tension steel area

b 0.003
d’
a c
= +
Figure 3.8

Compression reinforcement is provided to ensure ductile failure (i.e. tension steel must
As ¿ fy
yield). For the reason, therefore, the stress in tension ( is always equal for .
A 's ¿ by
On the other hand, stress of compression steel ( may either be or below
fy
. This stress must always be checked.

A 's = A s 2 A 's = A s 2 f y / f ' s


If the compression steel yields, then , otherwise , where
f's
is the stress of compression steel is given and is given by the following equation.
(see derivation in page 137)

c−d '
f ' s=600
Eq. 3-6 c

According to Section 410.4.3 of NSCP, for members with compression reinforcement,


ρb
the portion of equalized by compression reinforcement need not be reduced by the
As
0.75 factor. Thus, the maximum permissible is:
f'
Eq. 3-7 A s max =0.75 ρb bd + A ' ss y
f
ρb bd= A s 1
The expression 0.75 .

As A 's
STEPS TO DETERMINE AND OF ADOUBLY REINFORCED
MU
RECTANGULAR BEAM, GIVEN AND OTHER BEAM PROPERTIES

ρmax M u max
I. Solve for and
ρmax =0.75 ρb
'
0.85 f
β (600)
ρ =0.75 c 1

f y (600+f y)
max

¿
¿
ρ¿f y
ω= =¿
' ¿
f c

Mn max=φ f 'c ωbd (1−0.59 ω)


2

Mu≤φ Mn max
If design Singly Reinforced
(See Chapter 2)
M u > φ M n max
If design as Doubly Reinforced(proceed to step II)

II. M u > φ M n max

b
d
a ’
d d - d’
d
-a/2

= +
Figure 3.9

As 1
Solve for
As 1= ρmax bd

Mn 1 Mn 2 As 2
Solve for , and
Mn 1=Mn max

φMn 2=Mu−φ Mn max

¿ φ T 2 ( d−d )
'

φ Mn 2=φ As 2 f y (d−d )
'

¿
¿
As 2 =¿ ¿

III. Solve for the stress of compression steel


Solve for a and c:

C1 =T1
0.85
f ' c ab= A s1 f y 0.003
¿
a=¿ ¿
d’

a=β1 c
c
¿
c =¿ ¿ c–
d’
f 's / E s 0.003
=
c−d' c

c−d
f ' s=600
Eq. 3-8 '
c

f ' s≥ f y
If proceed to IV
f ' s <f y
If proceed to V

f ' s≥ f y f ' s=f


IV. then use y
(compression steel yields)
A 's =As 2

f ' s <f y , f's


V. then use (compression steel will not yield)
A 's =A fy
s2
f
's

Mn
STEPS IN FINDING OF A DOUBLY REINFORCED RECTANGULAR BEAM
AS A 'S ,
WITH GIVEN , AND OTHER BEAM PROPERTIES

There are three possible cases in doubly reinforced beams.

f s=f ' s =f y ¿
Case 1: Both tension and compression yields (
f s=f y , f ' s < f y ¿
Case 2: Tension steel yields and compression steel does not (

Case 3: Tension steel does not yield compression steel yields.


f s=f y , f ' s < f y ¿
(

Note: For doubly reinforced beams with effective depth d=250 mm or more, it is not
possible for both steels not to yield.

b
d’
a

d d - d’
d
-a/2

= +

f ' s=f
I. Assume compression steel yields ( y
)

¿
¿
As 2= A ' s =¿ ¿
¿
As 1= As− As 2=¿ ¿

II. Solve for a and c (assuming tension steel yields):


C1 =T1 f ' c ab= A s1 f y
0.85
¿
a=¿ ¿
a=β1 c
¿
c =¿ ¿
f s=600 d−c
Check: c f s≥ f y ,
If tension steel; yields, proceed to step III

f s <f y ,
If tension steel does not yield, proceed to step IV

III. Solve for the stress in compression steel


c −d '
f ' =600
s
c

f ' s≥ f y ,
If proceed to step IV
f ' s <f y ,
If proceed to step V

IV. f ' s ≥ f y ,compression steel yields


Since

Mn=Mn 1+ Mn 2

a
( )
¿ T 1 d− ' +T 2 (d−d )
2

a
( )
'
M n= A s1 f y d− + As 2 fy (d−d

)
2f ' s <f y ,
V. If compression steel does not yield.
c−d
f ' s=600 d
' c

a
c
d
From the stress diagram: d –
d d’
C1 +C 2=T
-a/2
0.85 f ' c ab+ A ' s f ' s= As f y
c −d '
β c b+ A ' 600 =Af
0.85 f
'
c 1
s y
c
s

c
Solve for by equation formula.

¿ −
c d'
Solve for f ' , f ' =600 =¿ ¿
s s
c

¿
Solve for β1 c =¿ ¿
a,a=

Mn :
Solve for
Mn=Mn 1+ Mn 2

( )
a '
¿ C1 d−2 +C 2 (d−d )

( )+ A
a ' ' '
M n=0.85 f ' c ab d−2 s f s (d −d )
VI. f s <f y but f 's =f y

T =C c +C 2 A s f s=0.85 f ' c ab+ A ' s f y


d−c '
A x 600 0.85 f
s = (β1 c)b+ A ' s f y
cc
¿
c = ; a ¿ β =¿
¿ = 1

Mn=Mn 1+ M c2

a
( )
¿ C1 d−' +C 2 (d−d )
2

a
( )
' '
M n=0.85 f ' c a b d− + As yf (

d−d )
2
DEEP BEAMS

According to Section 410.8 of the Code, beams with overall depth to clear span ratios
greater than 2/3 for continuous spans, or 4/5 for simple spans, shall be designed as
deep flexural members taking into account nonlinear distribution of strain and lateral
buckling.

Beams with web depth that exceed 900 mm have a tendency to develop excessive wide
cracks in the upper parts of their tension zones. According to Section 410..7 of NSCP, if
the depth of web exceeds 900 mm, longitudinal skin reinforcement shall be uniformly
distributed along both side faces of the member for distance d/2 nearest the flexural
Ask
tension. The area of skin reinforcement per meter of height on each side face
shall be

Eq. 3-9 Ask ≥1.0( d−750)


The maximum spacing of the skin reinforcement shall not exceed the lesser of d/b and
300 mm. Such reinforcement may be included in strength computations if a strain
compatibility analysis is made to determine stresses in the individual bars or wires. The
total area of longitudinal skin reinforcement in both faces need not exceed one-half of
the required flexural tensile reinforcement.

ILLUSATIVE PROBLEMS

DESIGN PROBLEMS

PROBLEM 3.15
a .305-mm wide rectangular beam has an overall depth of 560 mm. The beam is
reinforced with four 25-mm-diameter compression bars. The centroid fiber. Assume
and . Determine the following:
a) The balanced tension steel area and the nominal and ultimate balanced moment.
b) The maximum tension steel area and the nominal and ultimate maximum
moment.

SOLUTION

70 mm 305 mm

β1=0.85 a
d-d’420

4-25 mm
d=490

a) Balanced condition
mm

= +
600 d 600(490)
c b= 600+ f c b= 600+
415
y
cb=289.66 mm
70 mm
a=β 1 cb a=0.85(289.66)

a=246.21mm
f =600 f =600 289.66−70
sc
d−c sc
289.66
c

f sc =455 MPa> f y
yield
f sc =f
y

T 1 =Cc A s 1 f y=0.85 f ' c ab

A s 1 (415)=0.85 (29) (246.21)(305)

As 1=4,460 mm
2

T 2 =C ' s A s 2 f y =A 's f y

As 2=1,964 mm
2

Asb= A s 1 + A s 2=6,242mm
2
Balanced steel area,

M nb=C c ( d−a/ 2 ) +C' s ( d−d )


'

a
( )
'
M nb=0.85 Tf ' c a b d−' + A s f y (d−d )
2

M 246.21
nb
=0.85 (29) (246.21) (305)(490− )
2
M nb=1,021.4 kN −m

φM nb=0.90 (1,021.4)=919.24 kN −m
b) Maximum tension steel area:

According to Section 410.4.3, for members with compression reinforcement, the


ρb
portion of equalized by compression reinforcement need not be reduced by the
0.75 factor.
A s 1=0.75 A s 1 A s 1 max =0.75( 4,460)

As 1 max =3,345 mm2

As 2=1,964 mm 2

A s max = A s 1max + A s 2 A smax =3,345+1,964

As max =5,309 mm
2

C c =T 1 0.85 f ' c ab=A s 1 max f y

0.85 ( 29 ) ( a ) (305 )=3,345( 415)

a=184.7 mm

c=a/ β1 c=184.7 / 0.85

c=217.2 mm

c −d
f ' =600
' 217.2−70
f ' =600
s 217.2
c
s

f ' s=406.7 MPa<f y( will not yield )


a
( )
' '
M C d C d d
n max= c − + s( −
2
a
( )
) ' '
M n max=0.85 f 'c a b d− + As fs (d−d )
2
Mn max 184.7
=0.85(29 ) (184.7) (305)(490− )
2
+1964 (406.7 )(490−70)

M n max=887.45 kN−m

φMn max=0.90(887.45)

φM n max=798.7 kN−m

PROBLEM 3.16 (CE NOVEMBER 2009)

A reinforced concrete beam has width of 300 mm and effective depth of 460 mm. The
beam is reinforced with 2-28 mm compression bars placed 70 mm from extreme
concrete. Concrete strength and steel strength .
a) What is the balanced steel area considering the contribution of the compression
steel?
b) What is the maximum tension steel area allowed by the code?

SOLUTION
0.05
β =0.85− (35−30)=0.814
1
7

π 2 2
A'= (28) x 2=1,232 mm
s
4

a) Balanced condition considering compression steel:

600 d 600(460)
c b= c=b
600+ fy 600+345

c −d ' 292−70
f ' =600 f ' =600
s 292
c
s

f ' s=456 MPa>f y , thus f ' s=f y=345 MPa

C c +C s =T 0.85 f ' c ab+ A ' s f ' s= A s f y

0.85 (35) (0.814 x 292) (300)+ 1232(345)=A s (345)

As =7,384 mm
2

b) Maximum steel area:


For rectangular beams:
Cmax =0.75 cb =0.75(292)
Cmax =219.05 mm

a=β1 cmax=178.37 mm

c −d ' 219.05−70
f ' =600 f ' =600
s 219.05
c
s
f ' s=408 MPa> f y , thus f 's =f y=345 MPa

C c +C s =T 0.85 f ' c ab+ A ' s f ' s= A s f y

0.85 ( 35 ) (178.37 ) (300)

+1232(345 )=A s (345)

As =5,846 mm
3

PROBLEM 3.17

A rectangular beam has b=300 mm and d= 490 mm. Concrete compressive strength
and steel yield strength . Compressive steel if required
shall have its centroid 60 mm from extreme concrete fiber. Calculate the required
tension steel area if the factored moment is 620 kN-m.

SOLUTION
This is the same problem in Chapter

φM n max
2. Solve for :

600 d
c= =335.616 mm
b
600+ f y

ab =β1 cb=285.27 mm
a
Mn max =0.85 f 'c ab(d − )
2
M n max=0.85(27.6 ) (213.96 ) (300)(490−213.96 /2)

Mn max=576.76 kN−m

φM n max=0.90 (576.76 )=519 kN −m


Mu=620 kn−m> φM n max ,
Since the beam must be doubly reinforced.

b
d’
ca
Mn 1=Mn max=576.76
kN−m d – d – d’
= a/2+
Mu 0.90−576.76
Mn 2 Mn 2 = 620
= −Mn 1 Mn
φ
2 =112.13 kN −m

a=213.96 mm
A s 1 f y =0.85 f ' c a b A s 1 (276)=0.85(27.6 ) (213.96)(300)

A s 1=5456 mm
2
A s 1= A s max
Note:

f ' s:
Solve for
a
c= =251.71 mm
β1

c −d ' 251.71−60
f ' =600 f ' =600
s
c 251.71
s
f ' s=457 MPa>f y
Compression steel yields

Usef ' s=f y


M n 2=T 2 ( d−d ) 112.13 x 10 = As ( 276 )( 490−60)
' 6

As 2=945 mm 2

A s = A s 1 + A s 2=6401 mm
2
Tension steel area,
Compression steel:

c ' s=T2 A ' s f y=As 2 f y

A 's =As 2

A 's =945 mm2

PROBLEM 3.18
A rectangular beam has b=310 mm and d=460 mm. The beam will be designed to carry
a service dead load of 230 kN-m and service live load of190 kn-m. Compression
reinforcement if necessary will have its centtoid 70 mm from extreme concrete fiber.
f ' c=30 MPa f y=415 MPa.
Determine the required steel area. Use and

SOLUTION

β1=0.85

Mu=1.4 MD +1.7 ML Mu=1.4 (230 )+1.7 (190)

Mu=645 kN −m

φ M n max
Solve for :
cmax =0.75 cb
Note : For rectangular beams,

cmax 600 d
=0.75 =203.94 mm
600+ f
y

a=β1 cmax =173.35 mm

Mnmax=0.85 f 'c ab(d −a/ 2)

M n max=0.85(30 ) (173.35) (310)(460−173.35/ 2)

Mn max=511.58 kN−m

φMn max=0.90(511.58)

φMnm ax =460.42 kN −m

Mu=645 kN −m >φ M n max , compression steel is necessary


Since

b
d’
ca

= d – d – d’
Mn 1=Mn max=511.58 kN−m a/2+

Mu 645
Mn 2 Mn 2 = −511.58
= −Mn 1 0.90
φ
Mn 2=205.088 kN −m

c=cmax =203.94 mm

a=173.35 mm

Tension Steel:
T 1 =Cc A s 1 f y=0.85 f ' c ab

A s 1 (415)=0.85 (30) (173.35)(310)

As 1=3,302 mm
2

M n 2=T 2 ( d−d ) ( )
205.088 x 10 = A s 2 415 (460−70)
' 6

As 2=1,267 mm
2

A s =A s 1 + A s 2 A s =3,302+ 1,267

As =4,569 mm
2

Compression steel:

c −d ' 203.94−70
f ' =600 f ' =600
s 203.94
c
s

f ' s=394.06 Pa
compression steel does nt yield,

C ' s=T 2 A 's f ' s= A s 2 f y

A s ( 394.06 )=1,267( 415)


'

2
A 's =1334 mm
PROBLEM 3.19
A floor system consists of a 100-mm concrete slab supported by continuous T beam
with 9 m span, 1.2 m on centers as shown in Figure 3.10. Web dimensions, as
bw=280 mm,
determined by negative-moment requirements, are and d=500 mm.
Concrete cover is 70 mm from the centroid of the bars. The beam is subjected to a
f ' c=21 MPa, f y=415 MPa
maximum positive factored moment of 1080 kN-m. Use .
3
Unit weight of concrete is 23.5 kN/ m .
Calculate the required tension steel area at the point of maximum positive moment.
Using the tributary area method, what is the uniform service dead load acting on the
beam?
Calculate the uniform service live load acting on the beam.

A L=9m B L=9m
C

SOLUTION
f ' c=21 MPabw=280 mm
f y=414 MPa d=500 mm

β1=0.85 d =70 mm
'
M u max=1080 kN −m
Maximum factored moment,

bf :
Effective flange width,
1. L/4=9/4=2.25 m
b =16 (100)+ 280=1.88 m
2. 16t+ w
Soc =1.2 m
3.

bf =1.2 m
Use

φM n max
Solve for to determine if compression steel is
required. Balanced condition:

600 d 600(500)
c= c=
600+ f y 600+415

a=β 1 c a=0.85(295.57)

a=251.23 mm
= 1200mm

d =500mm
t=100

C a
z

z =
z
z=151.23 mm

A1=1200 x 100=120,000 mm
2

A 2=280 ( 151.23 )=42,345 mm


2

Acb =A1 + A2=162,345 mm


2

Maximum condition:

A c max =0.75 A cb Ac max =0.75(162,345)

Ac max =121,759 mm2> A1

= 1200mm
t=100
d =500mm

C a
z
A 2= Acmax − A2=121,759−120,000
A1
A2=¿ 1,759 mm2
T
A 2 =b w z 1,759=280 z
=280 mm
z=6.28 mm

y 1 =d−t / 2 y1=500−100/ 2

y1=450mm

y2=d −t −z /2 y2=500−100−6.28 /2
y2=396.86 mm

M n max=C 1 y1 + c2 y2 M n max=0.85 f c ( A 1 y1 + A2 y2)


'

M n max=0.85(21)(120,000 x 450)

+1,759 x 396.86
M n max=976.36 kN−m

φMn max=0.90(976.36)

φMn max=878.72 kN−m

Mu=1,080 kN −m> φM n max ,


Since the compression reinforcement must be provided.

=1
200mm d’=
a 70
d=500

500 mm

Z
mm

d’
d-
4 0
3

d’=
70

=820m
m

a=t + z a=100+6.28
a=106.28 mm
c=a/ β1 c=106.28 /0.85

c=125.04

c −d ' 125.04−70
f ' =600 f ' =600
s 125.04
c
s

f ' s=264.1 MPa< f y

Mn 1=Mn max=976.36 kN−m

A s 1= A s
max

T 1 =C1 +C 2 A s 1 f y=0.85 f c ( A1 + A2
'

)
A s 1 (415)=0.85 (21)(120,000+ 1,759)

As 1=5,237 mm 2

Mn 2
Mu
=Mn−Mn 1 Mn 2
= −Mn 1
φ
Mn 2 1080
= −976.36
0.90
Mn 2=223.64 kN −m

' '
M n 2=T2 ( d−d ) M n 2 =A s 2 f y( d−d )

223.64 x 10 =A s 2 ( 415 ) (500−70)


6

2
As 2=1,253 mm
A s = A s 1 + A s 2=6,490 mm
2
Tension steel area,

Compression steel area:


C ' s=T 2 A 's f ' s= A s 2 f y

' ( 264.1 )=1,253(415)


As
A 's =1,969 mm
2

b)

Dead load=weight of concrete:


Area=1.2(0.1)+0.28(0.47)=0.2516 m3
w c =γ c x Area

w c =23.5(0.2516)

wc=5.9126 kN / m →dead load

c) Uniform live load

7.6 m 7.6 m
5 32 mm3-3ø232
5mm5ø-
mm3-3ø2
- mm5ø-
A L=932mmmø
- 32 mmø B
L=9m C
145 kN-m 145 kN-m
202 N-m
k
Maximum positive moment (at midspan)
2
9¿
¿
w2u ¿
wu L
w= 1,080=¿
u
24
wu=320 kN /m

wu=1.4 wD + 1.7 wL 320=1.4 (5.9126 )+ 1.7 w L

wL=183.37 kN /m →liveload

INVESTIGATION (ANALYSIS) PROBLEMS

PROBLEM 3.20
The beam shown in Figure 3.11 is subjected to a maximum service dead load moment
of 230 kN-m. Determine the service live load that the beam can carry. Use
f ' c=20.7 MPa∧f y=345 MPa.

350
540 mm
600 mm

m6m0 mm 2-
28 mm

4-36 mm

Figure 3.11
SOLUTION
36
¿
¿π
A= ¿
s
4
2 2
28 ¿ x 2=1,232 mm
π
A'= ¿
s
4

b
d’
ca
all steel yield:
Assume – d – d’
= d
f s=f ' s =f y
a/2+

As 2= A ' s=1,232 mm2

A s 1= As− A s 2=2,840
2

mm

C c =T 1 0.85 f ' c ab=A s 1 f y


0.85 (20.7)a (350)=2,840(345)

a=159.1mm

a
c= =187.18 mm
β1
d−c
f 600
= f s =600 600−187.18
187.18
s
c
f s=1.323> f y tension steel yields

f s=600

'
c−d 187.18−60
f s=600 187.18
c
f s=407.7> f y
compression steel yields

Assumption is correct, all steel yield.

Mn=Mn 1+ Mn 2

a
(
M n=T 1 d− ' +T 2 (d−d )
2 )
a '

( − 2 )+
M A f d A f d d
n = s1 y s2 y( −

159.1
( )
)
M n=2,840 (345) 600− +1,232 (345)(600−60)
2

Mn=739.4 kn−m

φM n=0.90 (739.4 )=665.43 kN −m

φM n =M u =1.4 M D +1.7 M L 665.43=1.4 (230 )+ 1.7 M L


ML=202.02 kN−m

PROBLEM 3.21
A rectangular beam has the following properties:
Width, b=400 mm

Effective depth, d=620 mm


Tension bars, 3 pcs 25-mm-diameter
d’=70 mm
Determine the design strength of the beam and the safe service live load if the service
dead load is 320 kN-m.

SOLUTION
2 2
28 ¿ =6,158 mm
π
A =10 x ¿
s
4
2 2
25 ¿ =1,473 mm
π
A ' s =3 x ¿
4

Assume all steel yields:


As 2= A ' s=1,473 mm
2

As 1= As− As 2=4,685 mm
2

b
d’
ca

= d – d – d’
a/2+
0.85 f c a b= As 1 f 0.85 ( 22 ) a ( 400 )=4,685 ( 415 )
'

a=260 mm
a
c= =305.8
mm β1
d−c
f =600 =616.5 MPa>
s y ( yield )
f
c
c −d '
f ' =600 =463 MPa> f
( yield)
s y
c
All steel yields. Assumption is correct

Mn=Mn 1+ Mn 2

a
M n=T 1 d−
( 2 )+T 2 (d−d )

a
( −2 )
'
M A f d Ad d
n = s1 y + s 2( − )

M n=4,685 (415) (620−260/ 2)+1,473 (415)( 620−70)

Mn=1288.9 kN −m

φM n=0.90 (1288.9)=1,160 kN −m

φMn=Mu=1.4 M D +1.7 ML 1160=1.4 (320 )+1.7 M L

ML=419 kN −m

PROBLEM 3.22
A 12-m long rectangular reinforced concrete beam is simply supported at its ends. The
beam is provided with an addition support at the mid span. Width of beam is 300 mm
and the overall depth is 450 mm. The beam is reinforced with 25-mm-diameter bars,
four bars at the tension side and 2 bars at the compression side .Concrete protective
coverings is 70 mm form the centroid of the bars. Concrete strength and

steel yield . Use 0.75


Determine the depth of the compression block.
Determine the nominal moment capacity of the beam.
Determine the factored uniform load, including its own weight, the beam can carry.

SOLUTION
β1=0.85
300
f ' c=30 MPa
mm
f y=415 MPa 70 mm
2-25

450 mm
2
25 ¿
π mm
A =4 x ¿
s
310
mm
380

4
mm

As =1963 mm
2

2
25 ¿ 4-25
π
A ' =2 x ¿
s mm
4

A 's =982 mm
2 70 mm

Assuming all steel yields:


2
A s 2= A ' s=982mm
2
As 1= As− As 2=982 mm
Cc=T1 0.85 f 'c ab=As 1 f y

0.85 (30) a (30)=982( 415)


a=53.26 mm
a
c= =62.66 mm<70 mm
β1
compression steel does not yield

Assuming tension steel yields and compression steel does not.


T =C c +C ' s A s f y=0.85 f ' c ab+ A ' s f ' s

30 (0.85 c)(300)
1963 (415)=0.85¿
c−70
+982 x 600
c

c=98.87 mm

98.86−70
f ' =600 =175.17 MPa <f
s y
98.86

d−c
f =600 =1,706> f
s
c y ( yield )

a=β1 c=84.03 mm→answer ∈Part a

( )
a '
M n=C c d− 2 +C s ( d−d )

' ' '


Mn=0.85 f ' c ab+ A s f s (d −d )

Mn=270.58 kN −m→asnwer ∈ part b


φMn=0.90 Mn

φ Mn=243.53 kN −m

c) Maximum factored uniform load:

Factored
load,
By there-moment equation:
A B6 A 6 A2 a2 C
M AL +2 M B(L +
1 L 2)+ McL 2+ + ´ =0
1 1
1
L1

L2

M A=MC=0
6 A a´
3
w L
1 1 u 1
=
L1 4
6 A a´
3
w L
2 2 u 1
=
L2 4
+ 6+¿
¿3
6¿
¿
3
6¿
¿
wu ¿
wu ¿
0+2 MB ¿
wu=54.12 kN / m

PROBLEM 3.23 (CE NOVEMBER 2010)


A 6 meter long simply supported reinforced concrete beam has a width of 350mm and
an overall depth of 470 mm. The beam is reinforced with 2-28 mm compression bars on
top and 4-28 tension bars at the bottom, each located 70 mm from the extreme
concrete
fiber. Concrete strength , and steel yield strength
Determine the following:
a) Depth of compression blocks assuming both tension and compression steel
yields.
b) What is the ultimate moment capacity of the beam in kN-m?
c) Determine the additional concentrated live load that can be applied at midspan if
the dead load including the weight of the beam is 20 kN/m.

SOLUTION
L=6 m f ' c=20.7 MPa
Given :
b=350 mm f y=415 MPa

d=400 mm db=28

mm d ' =70 mm
2 2
28 ¿ x 4=2463 mm
Tension steel area π
A= ¿
s
4

2 2
28 ¿ x 2=132 mm
Compression steel π
area, A'= ¿
s
4
β1=0.85 φ=0.90

Assuming tension & compression steel yields:


2
As 2= A ' s=1232 mm
2
As 1= As− As 2=1232 mm
c c =T 1 0.85 f ' c ab=A s 1 f y

0.85 ( 20.7 ) a ( 350 )=1232( 415)

a=83 mm→answer ∈ Part a

c=a/ β1 c=83 / 0.85

c=97.64 mm

c −d ' 97.64−70
f ' =600 f ' =600
s 97.64
c
s
f ' s=170 MPa< f y

Thus, compression steel does not yield.

c−d
f ' s=600
Since compression steel does not yield, ' c

Assuming tension steel yields:


Cc+Cs=Ts
0.85 f ' c ab+ A ' s f ' s= As f y
c−70
0.85 (20.7) (0.85 c ) (350)+1232 x 600 =2463( 415)
c
c=130.08 mm
a=β1 c=110.6 mm

d−c
f 600
= f s =600 400−130.08
130.08
s
c
f s=1245> f y ( yield )

c −d ' 130.08−70
f ' =600 f ' =600
s 130.08
c
s

f ' s=277.11 MPa< f y

( −2 )
' '
M c d C d d
n = c + s( −

) a
( )
' ' '
M n=0.85 f ' c a b d− + As sf (d

M n=0.85
−d ) (20.7) (110.6) (350) (400−110.6 /2)+1232 (277.11) (400−70)
2
Mn=347.33 kN −m

φ Mn=0.90( 347.33)
Ultimate moment capacity=
φMn=312.6 kN −m →answer ∈ Par t b
Ultimate moment capacity=

3m 3m

M u =φ Mn=312.6 kN
−m
L=6m 2
MD L PL L
Mu=1.4 M D +1.7 M L =1.4 +1.7

M u
8 4
2
6¿
¿2
6¿
¿
PL ¿
20 ¿
312.6=1.4 ¿
PL=73.175 kN

PROBLEM 3.24
A beam section is shown in Figure 3.12. The beam will be subjected to a maximum
service dead load of 215 kN-m. What is the safe service live load moment for this
beam? Use

650 mm
360 m30mmm
25 mm

5- 25mm
Figure 3.12
8 -
30 mm

SOLUTION
β1=0.85
2
25 ¿
π
Tension steel, A =8 x ¿
s
4

As =3,927 mm2
2
25 ¿
Compression π
A ' =5 x ¿
steel,
s
4
360 m30mmm

650 mm
2
A 's=2,454 mm d
5- 25mm

25 mm

1
'
d =30+ =42.5 mm d
2 (25 )
8 - 25mm
Effective depth to extreme tension
bar:
1 30 mm
d =650−30− =607.5 mm
t
2 ( 25 )

Effective depth (to centroid of tension bar)


d=650−30−25−1/ 2(25)
d=582.5 mm

MD −215 kN −m

f ' c=21 MPa

f y=415 MPa

Assume all steel yields:


2
As 2= A ' s=2,454 mm
2
As 1= As− As 2=1,473 mm
C c =T 1 0.85 f ' c ab=A s 1 f y

0.85 ( 21 ) a ( 360 )=1.473(


415)

a=95.1 mm

c=a/ β1 c=95.1 /0.85

c=111.9 mm
c −d
f ' =600
' 111.9−42.5
f ' =600
s 111.9
c
s

f ' s=372 MPa< f y

Compression steel does not yield.

d’ 5-360
25 mm

mm
25mm a
c
d 5-
d-a/2d-d’
25mm
8 - 25mm
T
Note : There are two lawyersof tension bars which obviously yiel . Thus , thier

stresses are both set equal ¿ f y∧thier cgis located at thier geometric centroid .

T =C c +C ' s A s f y=0.85 f ' c ab+ A ' s f ' s

' c−d'
As f y =0.85 f (β1 c ) b+ A x 600 c
c 's

3,927 ( 415 ) =0.85 ( 21 ) ( 0.85 c ) (360)


c−42.5
+2,454 x 600
c
c=122.38 mm
c −d
f ' =600
' 122.38−42.5
f ' =600
s
c 122.38
s

f s=391.64 MPa< f y

a=β 1 c a=0.85(122.38)

a=104.03 mm

a
( )
'
M n=C c d− ' +C s (d −d )
2
a ' '

( − 2 )+
M 0.85 f ' ab d A'f d d
n = c s s( − )

104.03
M =0.85 (21) (104.03 ) (360)(582.5− )
n
2
+2,454 (391.64 )(582.5−42.5)

Mn=873.68 kN

−m φMn=0.90(

873.68)

φMn=786.31 kn−m

M u=φ M n Mu=1.4 M D +1.7 M L

786.31=1.4 (215)+1.7 (M L )

ML=285.5 kN −m

PROBLEM 3.25
A beam section is shown in Figure 3.13. The beam will be subjected to a maximum service
dead load of 360 kN-m. What is the safe service live load moment for this beam? Use

320 3m0mmm
30 mmd’

650 mm
28 mm
650
mm
2- 2 - 25mm
β1=0.85

28 mm
25mm d
Fi2g8 ¿u re 2

3.13
Tension steel, π
A =10 x ¿ 10 - 28mm 10 - 28mm
s
4
30
A s =6,158 mm
2
mm 30 mm

2
25 ¿
π
Compression steel, A ' =2 x ¿
s
4

A 's =982 mm
2

d =30+1/ 2 ( 25 )=42.5 mm
'

Effective depth (to centroid of tension bars)


d=650−30−28−1/ 2(28)
d=578 mm

MD =360 kN −m

f ' c=21 MPa

f y=415 MPa

Assume all steel yields:


A s 2= A ' s=982mm
2

As 1= As− As 2=5,176 mm2

C c =T 1 0.85 f ' c ab=A s 1 f y

0.85 ( 21 ) a ( 320 )=5,176( 415)

a=376.04 mm

c=a/ β1 c=376.04 / 0.85


c=442.4 mm

c −d ' 442.4−42.5
f ' s =600 f ' s=600
c 442.4

f s=222 MPa< f y

Tension steel does not yield.


Note :
Thd e 320 myers of tension bars which have different stresses less than f .
5 e m m a y
a re c
r ’ 2
255

d’
d-
a/2
- tw o la

d-
mm
Thus, thier cg isnot located at thier geometric centroid .
d1=6
m d
80m- 28mm
1 50−30−14=606 mm
d2=650−30−28−28−14=550 mm

2 2
28 ¿ =3,079 mm
Ast 1 π
=5 x ¿
st
=A 2 4
T1 +T 2=Cc+C 's

Ast 1 f s 1+ Ast 2 f s 2=0.85 f 'c ab+ A ' s f y


d1 −c d2−c
Ast 1 600 + st
2
600 =0.85 f ' ab+ A 's f y
c c
A c

606−c 550−c
3,079 x 600 +3,079 x 600
c c
¿ 0.85 (21) (0.85 c ) (320)+982 (415)
c=363.9 mm

a=β1 c=309.29 mm

d1 −c
fs = 600 606−363.9
1
=600 s1
363.9
c
f
f s 1=399.25 MPa<f y

d2−c =600 550−363.9


s2
fs 363.9
=600 f
2
c f s 2=306.9 MPa<f y

c −d ' 363.9−42.5
f ' =600 f ' =600
s 363.9
c
s
f ' s=530 MPa> f y

a=β1 c a=0.85(345.4)

a=301.2mm

Solve for d:
T1 =Ast 1 f s 1 T 1=3,079(399.25)

T1 =1,229.2kN

T2 =As 2 f s 2 T 2=3,079 (306.9 )

T2 =944.9 kN

T x d=T1 x d1 +T 2 x d2

2,174.1 d=1,229.2 ( 606 )+ 944.9(550)

d=581.66 m

a
( )
' '
M n=c c d − +C s ( d−d

) a
(
M2 n=0.85 f ' c a b d−'
2 )+ A ' f ( d−d )
s y

M n=0.85 (21) (309.29 ) (320)(578−309.29/ 2)

+982 ( 415 ) (581.66−42.5)

Mn=974.07 kn−m

φMn=0.90( 947.07)

φMn=876.65 kN −m

Mu=φ Mn Mu =1.4 MD +1.7 M L

876.65=1.4 (360)+ 1.7( M L )

ML=219.21 kN−m

PROBLEM 3.26
Calculate the design flexural strength of the T-beam shown in Figure 3.14. Use

=600mm
25mm

5
m
m
3-23mm t=100mm
Figure 3.14 10mm
10-2 stirr=u3p90mm
25mm
20mm
=300mm

.
SOLUTION
β1=0.85
2 2
25 ¿ =4,909 mm
π
A s =10 x ¿
4
2 2
22 ¿ =1,140 mm
π
A 's=3 x ¿
4
( )
Flange area, A f =600 110 =66,000 mm
2

Assume all steel yields:


d=110 +390−20−10−25−1/ 2 (25)=432.5 mm

d =25+10+1 /2 ( 22 )=46 mm
'

2
As 2= A ' s=1,140 mm
2
As 1= As− As 2=3,768 mm
Area of compression concrete:
C c =T 1 0.85 f ' c A c = A s 1 f y

0.85 (27) Ac=3,768(350)

Ac=57,468 mm2< A f therefore a<

t A c =ab f 57,469=a(600)

a=95.8
mm<t c=a/ β1 c=95.8/ 0.85

c=112.7 mm

c −d ' 112.7−46
f ' s =600 f ' s=600
c 112.7

f ' s=355 MPa> f y ( yield )

=600mm 110mm
m 3- 25 m
=390m

a/2

2
23 mm d’
d-

d 10mm
d-

10-25mm stirrup
25mm 20mm
m

=300mm
d−c
f =600 432.5−112.7
=600
f
s 112.7
c
s
f s=1,703> f y ( yield )

Verify if the upper layer of tension steel yields


d 2=d −1/ 2(25)−1/ 29(25 )=407.5 mm
d2−c
fs ( yield)
2
=600 =1,567 y
c
MPa>f

All steel yields, assumption is correct:


a
( )
'
M n=C c d− +C s (d −d )
2
a
( )
' '
M n=0.85 f ' c a b f d− + As yf (d−d

)M n=0.85 (27) (95.8) (600)432.5−46 ¿


2
+1,140(350)(432.5−46)
Mn=661.5 kN −m

φMn=0.90( 661.5)

φMn=595.4 kN−m

PROBLEM 3.27
Calculate the design flexural strength of the T-beam shown in Figure 3.15. Use

=600mm

25mm
t=100mm
2-
22mm
Figure 10mm
3.15 stirru p
= 3
90mm
10-28mm
25mm
20mm
=315mm
SOLUTION
β1=0.85
2 2
28 ¿ =6,158 mm
π
A s =10 x ¿
4
2 2
22 ¿ =760 mm
π
A 's=2 x ¿
4

A f =600 ( 100 )=60,000 mm


2
Flange area,

Assume all steel yields:


d=100+390−−20−10−28−1 /2 (25)=419.5 mm

d =25+10+1 /2 ( 22 )=46 mm
'

As 2= A ' s=760 mm
2

As 1= As− As 2=5,397 mm2

Area of compression concrete:


C c =T 1 0.85 f ' c A c = A s 1 f y

0.85 (25) Ac=5,397(345)


2
Ac=87,626 mm > Af therefore a>t

Ac= Af + Aw 87,626=60,00+ Aw
2
Aw=27,626 mm
A w =b w z 27,626=315 z

z=87.7 mm

a=100+ z=187.7 mm

c=a/ β1 =220.83 mm

c −d ' 220.83−46
f ' =600 f ' =600
s 220.83
c f ' s=475
s MPa> f y
( yield)

Вам также может понравиться